Этого треда уже нет.
Это копия, сохраненная 9 сентября 2019 года.

Скачать тред: только с превью, с превью и прикрепленными файлами.
Второй вариант может долго скачиваться. Файлы будут только в живых или недавно утонувших тредах. Подробнее

Если вам полезен архив М.Двача, пожертвуйте на оплату сервера.
Тред тупых вопросов №106 GODDARD EDITION 480719 В конец треда | Веб
Тред вопросов о жизни, Вселенной и всём таком.

Спрашиваем то, за что в других местах выдают путёвку в биореактор. Здесь анонимные учёные мирового уровня критически рассмотрят любые гениальные идеи и нарисованные в Paint схемы.

Предыдущий тут: >>475419 (OP)
https://2ch.hk/spc/res/475419.html (М)

Q: Можно быстрее?
A: Можно упасть в пузырь альбукерке, наса уже почти надула его.

Q: Я начитался охуительных историй про уфологию, че делать, нам жопа?
A: Да, тебе жопа, можешь сгонять в зогач или куда оттуда пошлют.

Q: Что будет с человеком в вакууме без скафандра / если он упадет на черную дыру / попробует ступить на поверхность газового гиганта/солнца?
A: Он умрёт.

Q: Почему бы не привязать ракету к воздушному шару или стартовать с горы?
A: Космос - это не как высоко, а как быстро, большая часть энергии ракеты уходит на разгон вбок.
Подробнее тут https://what-if.xkcd.com/58/ (английский) https://chtoes.li/orbital-speed/ (перевод)
2 480724
Ебанный стыд...
Во-первых, Алькубьерре.
Во-вторых, не упасть, а создавать вокруг корабля изнутри (иначе кина не будет).
В-третьих, НАСА искривляет пространство на десятимиллионную часть, контролируя это сверхточными интерферометрами, до самого варп-привода здесь - как до Антарктиды раком.
3 480726
Вопрос, наверное, и в самом деле тупой.
Можно ли это тред разделить на два: для тупых вопросов по астрономии и астрофизике и для тупых вопросов по космонавтике и ракетостроению? Совсем разные темы же.
4 480729
>>0726
Нет, скорость тредов вполне нормальная, чтобы успевать и то, и другое.
5 480735
Как с помощью гравитационных манёвров достичь ОЧЕ больших скоростей? Если уж не скорости света, то хотя бы субсветовой. Хотя бы половину от Ц, хотя бы треть...
6 480736
>>0735
Пролетать очень близко от центра гравитирующего тела, то есть проводить манёвр у чёрной дыры.
7 480759
>>0726
Может твой мозг напополам поделить, умник?
8 480778
>>0735
>>0736
Этого мало, нужно еще что бы эта ЧД сама двигалась с околосветовой скоростью в той системе отсчета, в которой мы меряем скорость корабля. Иначе вся скорость, которую корабль наберет при падении к дыре израсходуется в процессе подъема.
9 480782
Поясните для довнича на пальцах, как работают холодильники.
10 480783
>>0782
Отбирают тепло у помидоров и передают его окружающей среде. В них течёт жидкость по трубкам. Расширяясь и испаряясь в той части трубок, которая в холодильной камере, она отбирает тепло у окружающей среды, затем её сжимают обратно, она нагревается и отдаёт тепло наружу (это происходит на задней панели холодильника).
11 480785
>>0783
А манометры на холодильных установках показывают типа давление в испарителе и в компрессоре? Видел просто, что их там 2 штуки и один постоянно показывает мало, а второй много.
12 480796
>>0724

>НАСА искривляет пространство на десятимиллионную часть


С помощью чего?
13 480797
>>0783
Обязательно фреон нужен? Просто водой можно?
14 480798
>>0783
Реально ли сделать из говна и палок холодильник, способный замораживать до -70 градусов?
15 480800
>>0796
Тяжёлой гири, вероятно.
16 480801
>>0800
Раскручивают её на центрифуге или шо?
17 480805
>>0785
Ну да видимо, ну то есть наверное в трубке с холодным паром и в трубке с горячей жидкостью.

>>0797
Водой нельзя. Можно газом, без фазового перехода (воздухом например), но это не так эффективно. Там суть в том, что у жидкости очень низкая температура кипения, и забираемое тепло расходуется на фазовый переход.

>>0798
Ну, можно попробовать что-нибудь придумать. Может быть можно использовать машину Стирлинга или ещё какую-нибудь простую тепловую машину. Ещё есть эффект Пельтье, за счёт которого можно делать холодильник просто на электричестве. Гугли элемент Пельтье.

/spc/ - бытовая и оргтехника
18 480806
>>0805

>Ещё есть эффект Пельтье, за счёт которого можно делать холодильник просто на электричестве.


Хуйня без задач, больше, чем 30 градусов разницы температур между двумя поверхностями достичь практически нереально.
19 480807
>>0719 (OP)

>GODDARD EDITION


Конгрива дождёмся?
20 480811
>>0806

>больше, чем 30 градусов разницы температур между двумя поверхностями достичь практически нереально


А если к тёплой стороне одного элемента Пельте прилепить охладители двух других элементов Пельтье, а к их нагревателям — охладители от ещё четырёх элементов Пельте? Разность температур (при должной теплоизоляции) должна будет составить 90 градусов, так?
21 480812
22 480835
>>0807
Можешь сам перекатывать имени кого захочешь.
23 480874
>>0759
а так можно?
24 480876
>>0874
Можно. Фактически, он и так уже поделён, полушария могут выполнять одни и те же функции, и дублируют друг друга. Люди нормально живут с удалённым полушарием, особенно если удаление произошло в детстве (тогда благодаря большой нейронной пластичности мозг хорошо адаптируется к работе одним полушарием).
25 480929
На каком музыкальном инструменте можно сыграть на поверхности Марса?
26 480930
>>0929
На кожаной флейте.
27 480931
>>0929
На любом, кроме духовых.
28 480936
Почему перестали строить и юзать дирижабли?
29 480938
>>0936
Строят и юзают, например метеозонды.
30 480944
>>0931
А музыку мы услышим?
31 480947
>>0944
А схуяли нет? Там есть и атмосфера,а волны будут по твоему телу и земле лол передаваться
32 480951
>>0931
Да и на духовых можно. если дырку в скафандре провертеть
33 480972
>>0944
Это уже второй вопрос.
Так-то ты мало что услышишь, из-за низкой плотности марсианского воздуха.
34 480976
У меня вопрос на стыке с литературной фентези.
Где-нибудь в каких нибудь произведениях хоть бы чем-нибудь НАУЧНО объясняется, почему солнце и фазы луны влияют на магическую силу?
35 480977
>>0976
Это не стык с фентези, а просто вопрос про фентези. Тебе на другую борду.
36 480981
Q
37 480996
Бывают галактики из двух-трех звезд, или к такой мелочи понятие галактика не применяется?
38 480998
>>0996
Самая мелкая около тысячи
39 481009
>>0998
А в принципе могут такие микроскопические галактики быть? Дыры же и нейтронные сперва в уме придумали, а потом нашли их.
40 481010
>>0996
Нижнего предела в номенклатуре нет, главное чтобы скопление говн (не только звезд!) не было в составе другой галактики. Хотя в краевых случаях, думаю, придумают какой-нибудь особенный термин, как с коричневыми карликами. Систему из двух звезд, свободно летающую вне галактик назовут не галактикой, а просто двойной системой, например. Надо понимать что это лишь номенклатура для удобства каталогизировани, на реальность никак не влияющая. По факту говна, летающие в космосе, гравитационно связаны самыми причудливыми способами, и образуют самые ебанутые феномены, которые точной классификации часто не поддаются.

>>0998
Это самая мелкая задетекченная.

>>1009
Могут конечно, в межгалактическом пространстве много всякого говна летает, наверняка полно и мелких гравитационно связанных систем, которые по текущей номенклатуре будут галактиками считаться.
41 481015
>>0938
Да есть и тру-дирижабли для туристов.
42 481021
Как можно представить гравитационные волны, чтоб было понятно на уровне: атом и вокруг него электроны летают, похуй, что там какое-то электронное облако? Само понятие волна представляется, как расходящиеся круги, а не притягивающее в себя нечто.
Не может же она идти задом-наперед.
43 481043
Почему вблизи черных дыр нет стабильных орбит?
44 481050
>>1021
Чем не нравится представление пространства как резинового листа, а гравитации как вмятин на нем? Очень хорошая аналогия.

https://www.youtube.com/watch?v=MTY1Kje0yLg

В такой аналогии гравитационные волны легко наблюдаются при движении вдавленностей по листу, любое переменное ускорение создает расходящуюся в стороны рябь.

https://www.youtube.com/watch?v=rDVyuCEPmVs
45 481077
Сейчас видел интересное явление. Далеко, над горизонтом летел какой то яркий объект, прям очень ярко светил. Свет как от сварки или как от осколка стекла в солнечном свете. При этом по размерам очень маленький, как звезда, точка в общем. Летел параллельно горизонту на юго-западе. С запада на юг.
Что это может быть? Посмотрел во вспышках иридиумов, но там его нет.
46 481080
>>1077
Самолёт, метеор, какой-то другой спутник, оптический эффект, близкий объект, который ты принял за далёкий, корабль Надсмотрщиков с Тау Кита.
47 481086
>>1080
Вряд ли это самолет. Следа не было, да и слишком быстро летел для такого расстояния до меня.
Точно не метеор, метеоры так долго не летят.
Видимо какой-то спутник.
48 481095
>>1077
Свет Венеры отразился в болотном газе.
1380997729795.jpg37 Кб, 345x490
49 481111
В моем больном разуме возник вопрос. Что будет с Солнцем, если его "потушить" в огромном резервуаре с водой? Резервуар этот, наверное, должен быть размером с саму Солнечную систему или даже больше. Но если представить, что Солнце опустили в него, словно бы мячик в бассейне под воду, то что с нашей любимой звездой произойдет? Она превратится в окаменелость? Испарится без следа? Рассыпется на какие то фрагменты? Вариант с тем, что испарит воду не рассматривается, потому что тогда берем столько воды, чтобы не испарило.
50 481115
>>1111
В твоей задаче изъян в том, что бассейн воды с солнечную систему - сама по себе очень большая масса. Если основная часть материи в солнечной системе скомпонована в веществе +/- с плотностью воды, то представь, что вся пустота системы вдруг стала не полой, а стала целым бассейном. Тут интереснее что с этим бассейном будет. Если же ты через йоба портал начнешь лить на макушку солнца столько, сколько оно сможет сожрать, то потухнуть не потухнет конечно. Начнет аккумулировать на себе водород, и когда достигнет критической массы, ебнет сверхновой, разметая излишки вещества.
51 481116
>>1111
Ты внезапно только что ядерный реактор. Только вместо ТВЭЛов у тебя звезда.
52 481120
>>1111
Ничего не будет. Солнце работает от термоядерной реакции, которая возможна из-за гравитационного сжатия. Добавляя воды, ты лишь подкидываешь Солнцу массы и топлива.

>Вариант с тем, что испарит воду не рассматривается, потому что тогда берем столько воды, чтобы не испарило.


Такое количество воды само превратится в звезду.
53 481126
>>1120

>Такое количество воды само превратится в звезду.


Морская водяная звезда?
54 481128
>>1120
>>1126
Такое количество воды сколлапсирует в черную дыру.
55 481131
>>478503
Так, падажжи. Значит если наблюдатель будет находиться на расстоянии, допустим, нескольких световых секунд от горизонта событий, то для него всё произойдёт так же быстро, как и для падающего за горизонт? Ну получается падение за горизонт событий для наблюдателя ничем не отличается от падения на любое другое небесное тело.
56 481144
Может ли быть такое, что две планеты примерно одинакового размера приливно захвачены друг к другу и одновременно к звезде, вокруг которой они обращаются? То есть с поверхности одной планеты другая планета и звезда всегда в одном положении?
57 481145
>>1050

>ем не нравится представление пространства как резинового листа, а гравитации как вмятин на нем? Очень хорошая аналогия.


Потому что 2D поверхность листа вложена в 3D пространстве, а ИРЛ гравитация распространяется в 4D пространстве-времени которое никуда не вложено.
58 481147
>>1144
Чисто стереометрически это возможно только если планеты вращаются в плоскости, перпендикулярной направлению на звезду. А, значит, эта плоскость тоже должна вращаться с периодом в год. Не уверен, возможна ли такая конфигурация физически, но кажется нет: вроде как ничего не может заставлять эту плоскость вращаться с такой скоростью. То есть, как мне кажется, даже искусственно такое не создать.
59 481151
>>1145

>никуда не вложено.


У тебя настало хорошее время, чтоб принять струю на твой ротешь предоставить присутствующим пруфы тобою заявленного.
60 481154
>>1151
Пруфы отсутствия? Сразу после того, как предоставишь пруфы обратного?
61 481171
>>1126
Водородная. Вода это молекулярное соединение. В достаточных количествах гравитация сожмет ядро до такого давления, что вспыхнет реакция, в которой уже никаких молекул существовать не будет, одна плазма. Водород из воды будет гореть, кислород хуй знает что будет делать.

>>1128
Ну если и после этого продолжать добавлять - то может и в черную дуру, да.
62 481211
>>1171

>Ну если и после этого продолжать добавлять - то может и в черную дуру, да.


>


А массы воды объемом с СС не хватит? Если даже взять объем воды размером с Солнце, это уже будет ~18 солнечных масс, что весьма немало.
63 481219
>>1171

>кислород хуй знает что будет делать


Тоже гореть, это все еще достаточно легкий элемент. В достаточно массивных звездах горит все, вплоть до железа.
64 481221
>>1211
Хватит с огромным запасом. ЧД с массой Солнца имеет радиус около 3 км, а радиус ЧД прямо пропорционален массе. Расстояние до Урана (а СС гораздо больше) - 3 10^12 м, радиус Солнца - 7 10^8 м, плотность Солнца примерно в 1,4 раза выше чем у воды при н. у. То есть, шар воды радиусом в орбиту Урана будет весить примерно как 55 миллиардов Солнц. Его радиус Шварцшильда будет около 1,6 10^15 м, то есть уже этот шар будет ЧД.
65 481240
Сап астрономы
Вот вам тупой вопрос от меня
Давно уже беспокоит парадокс сверхсветовых путешествий
Вот допустим, я весь такой астронавт, лечу в жопу Вселенной выше скорости света, долетаю до нужного места за месяц например
С чего ради на Земле проходит охулиард лет, а не тот же месяц?
66 481241
>>1240

>выше скорости света


Это тебе в магач.
67 481247
>>1241
ОК, пусть СО скоростью света
Поняли ведь все, что я имею в виду
68 481248
>>1247

> СО скоростью света


В магач пройдите.
69 481253
>>1145
Топологически, Вселенная плоская. Да, это "лист", скорее всего замкнутый сам на себя - чувствительности Планка и WMAP не хватило, но две будущие миссии обещают пруф. Этот лист можно воспринять как плоскость, но с одним условием - в каждой точке этого листа можно создать трехмерную сферу почти произвольного размера (ограничено скоростью создания - c). Наш трехмерный мир есть совокупность таких сфер.
Это если грубо и на пальцах объяснить господствующее описание топологии В на сегодняшний день. Планк летал по сути только ради проверки этой гипотезы - поиски квазаров и пр были побочной задачей
70 481259
Может ли достаточно массивная/большая черная дыра всосать в себя целую галактику? И если да, то куда эта галактика со всем своим содержимым денется? Например если вместо Малого Магелланового Облака будет примерно такая же по размерам ЧД, то что дальше?
71 481264
>>1253

>Да, это "лист", скорее всего замкнутый сам на себя - чувствительности Планка и WMAP не хватило, но две будущие миссии обещают пруф



Чиво блядь? WMAP как раз хватило, чтобы показать, что никакого замыкания в видимой вселенной практически наверняка нет. Если бы настоящий размер вселенной был меньше видимого диаметра, то на «краях» начались бы повторения, но никаких повторений обнаружено не было.

https://arxiv.org/abs/1303.4004
72 481266
>>1259

>Может ли достаточно массивная/большая черная дыра всосать в себя целую галактику?


Дыры не сосут и не втягивают как-то особо. Если галактика в нее влетит - то да, ей пизда.

>И если да, то куда эта галактика со всем своим содержимым денется?


В дыру. Что происходит в дыре - хз. С точки зрения чистой ОТО там сингулярность, в которой собрано все вещество в одной точке. Но эффекты квантмеха тут явно будут влиять значительно. Проблема в том, что квантмех с ОТО не стыкуется, а тут нужно их как-то совместить.

>Например если вместо Малого Магелланового Облака будет примерно такая же по размерам ЧД, то что дальше?


По "габаритам" или по массе? Если по массе - ничего особенного, никто и не заметит. Все соседи будут лететь по тем же траекториям.
Black Hole Comparison.webm16,9 Мб, webm,
640x360, 4:09
73 481269
>>1259

>Может ли достаточно массивная/большая черная дыра всосать в себя целую галактику?


Может, и даже не одну. https://en.wikipedia.org/wiki/Phoenix_Cluster Правда условия для этого нужны специфические, поэтому этого почти не происходит.

>И если да, то куда эта галактика со всем своим содержимым денется?


В сингулярность. Наверное. Потому что хуй знает что там под горизонтом. Да и он не знает.
74 481270
>>1269

>Правда условия для этого нужны специфические, поэтому этого почти не происходит.


>почти


То есть прецеденты были? Или это просто фигура речи?
75 481275
>>1270
Любая СМЧД в себя уже всосала дохуя говн, потому она и СМ, собственно. Прецедент разжирания ЧД до массы парочки небольших галактик - выше по ссылке. Чтобы она прямо разрослась до ебанутых размеров - нужно чтобы с ней сталкивалось дохуя всего, или хотя бы очень рядом пролетело. А этого обычно не происходит, например вокруг Стрельца А* в центре нашей галактики обращается дохуя звезд с очень малыми перицентрами, та же S2 год назад просвистела на 7600км/сек в 120 а.е. от черной дыры и осталась целой.
76 481276
Насчет формы Вселенной.
Верно ли сказать, что после БВ мы существуем на плоскости раздувающегося шарика ?
То есть, вся наша Вселенная это "ударная волна" и ошметки энергии, превратившейся в материю ? А звезды - остывающие искры от БВ.
77 481278
>>1211
https://youtu.be/qpG6X919ayU
Вот тут уже смоделировали
78 481280
>>1276
Теория БВ после последних данных с Хаббла под большим вопросом.
79 481283
>>1280
А конкретнее, что нового открылось ?
80 481286
>>1283
Прямой контакт с космосом в голове предыдущего шизика.
81 481295
>>1211

>Если даже взять объем воды размером с Солнце, это уже будет ~18 солнечных масс


Из чего такой вывод? Из молекулярной массы? Ну нихуя экспертиза!
82 481296
>>1264
Да, WMAP показал, что скорее всего (2 сигмы кстати) реальный размер В больше наблюдаемого. И все. Планк улучшил этот результат до 6 сигма. Но главное - мультипольная анизотропия. Само ее существование показывает наличие замыкания, поэтому сейчас и идёт по сути перебор из вариантов топологий пространства Минковского (евклидова для нашего обсуждения) с положительной, но почти единичной плоскостностью. Вон, додекаэдры уже отвергли.
Главный мой посыл в том, что сама по себе инфляционная модель предполагает наличие таких свёрток, и даже если видимый радиус меньше, чем размер "ячейки", мы все равно должны видеть повторения в виде сфер первичного возмущения. Проще, мы скорее всего видим свой затылок, но не можем его узнать, так как видим его 13ккк лет назад. Собственно, нам не хватает чувствительности, чтобы видеть повторения в диапазоне нано-фемтокельвин. Хуево охлаждаем зеркало.
83 481297
>>1240
Парадокс сверхсветовых путешествий действительно скорее в /сайфай. Ты, скорее, подразумевал парадокс близнецов.

>С чего ради на Земле проходит охулиард лет, а не тот же месяц


Такая специфика ткани мироздания в этой вселенной. Равно такая же почему масса притягивает массу. У тебя не возникает когнитивных искажений, когда ты видишь падающее с ветки на землю спелое яблоко. Потому что это неотъемлемая часть бытовой повседневности. Если бы мы сейчас жили в ситуации, когда допустим половина населения Земли улетела 500 лет назад в космической карете, а сейчас вернулась, постарев на пару месяцев, то после волн социальных нестабильностей все бы привыкли. Возвращенцы получили бы какую-нибудь забавную сленговую погремуху в духе "старообрядцев". Кучи общественных организаций пилили бы бюджеты на культивацию терпимости и борьбу с новой ксенофобией - с угрюмыми землянами, которые не рады гостям из прошлого. И все это также стало бы обыденностью, и никто бы не задавался вопросами с философским оттенком "а почему так" чаще, чем задается подобными вопросами в адрес падающего яблока?
84 481306
Что из себя представляет коричневый карлан WISE 1828+2650 с температурой поверхности -23° – +127°? По своим размерам он как jewпитер. На эту звезду можно высадиться, или она как газовый гигант не имеет твердой поверхности?
85 481308
>>1306
Как Юпитер, только погуще.

Вообще верхний предел твердых каменистых планет около ~5 масс Земли, более тяжелая хуйня практически не будет терять водород и гелий, как их теряет Земля, и они продолжат накапливаться, пока не образуется газовый гигант.
86 481334
>>1295
А как ещё ты предлагаешь оценить массу водяного пузыря объёмом с Солнце?
87 481341
>>1334
Не натягивать сов на глобус, принимать несжимаемой, не учитывать всякие там модификации льда в ядре и просто считать по плотности? Всяко точней выйдет.
88 481343
>>1308
То есть из венеры не получится газового гиганта?
89 481349
>>1341
Явно выйдет хуйня из-за такой ебической массы.
90 481352
>>0929
Пока что играют только на ударных об поверхность.
91 481362
>>1334
>>1341
Солнце имеет определенный объем не только потому что содержит столько-то водорода. Это дохуя сложная система, которую пидорасит изнутри магнитным полем, термоядерными реакциями, довлением излучения и чем только не.
92 481384
Может ли пространство сжиматься быстрее скорости света? Если да, почему при этом не нарушается принцип причинности?
93 481387
>>1384

>Может ли пространство сжиматься быстрее скорости света?


Ну расширяться точно может. Достаточно удаленные объекты от нас удаляются куда быстрее скорости света, именно за счет расширения пространства.

>Если да, почему при этом не нарушается принцип причинности?


Тут вопрос другой: а почему он должен нарушаться? Приведи пример.
94 481388
>>1384
потому что расширение идет из каждой точки пространства
95 481414
Смотрел только что лекцию Сагана 1977 года, он там говорит, что в лаборатории воссоздали марсианские условия и некоторые микроорганизмы смогли приспособиться и даже размножаться при наличии воды. А на Марсе такие эксперименты проводились? Я только про цветочки на Луне слышал, к сожалению.
96 481416
>>1414
Нет. Как раз потому, что микроорганизмы теоретически способны выжить снаружи, никто такие эксперименты не проводит, наоборот, посылаемые аппараты стараются максимально стерилизовать перед стартом.

Еще не хватало затереть возможные следы ранней марсианской жизни какой-нибудь земной хуйней.
97 481422
>>0719 (OP)
Посоны, когда мы сами сможем погулять по Луне?
За адекватные суммы и требования к здоровью?
Сколько лет ещё для этого надо? 30? 50?

Или письками снова все померяются и заглохнет опять надолго?

Почему не могут взлетать с меньшим ускорением? Не 3-4g, а 1.5g? Тогда ведь и более хрупкие штуки можно запустить и меньше требований к здоровью специалистов.
98 481427
>>1422
На здоровье давно похуй. Посмотри на космонавтов, в последнее время дохуя жирных увальней пускают на МКС, раньше такой хуйни не было.
99 481429
>>1427
Жир = здоровье. Всякие кочкобляди имеют дохуя болячек, пока жиробоги наслаждаются жизнью.
100 481431
>>1429
Репорт жирного
101 481450
>>1422

>Почему не могут взлетать с меньшим ускорением? Не 3-4g, а 1.5g?


Потому что больше топлива нужно будет. Причем не чуток, а в разы.
102 481452
>>1422

>Почему не могут взлетать с меньшим ускорением? Не 3-4g, а 1.5g?


Взлетают РКН как раз с ускорением обычно 0,2-0,6 g. Но по мере расходования топлива, разумеется, уменьшается масса, а двигатели слишком сильно дросселироваться не умеют.
103 481455
>>1416
Тупая политика. Если человечество себя выпилит, то хоть это разнесет жизнь по солнечной системе.
Ну, кстати, первые марсоходы не стерилизовали, так что норм.
104 481456
>>1452

>0,2-0,6 g


это медленное падение, долбоеб
105 481467
>>1456
Ракеты взлетают вверх. Кто тут долбоёб-то?
106 481468
>>1467
С ускорением 1,5g они взлетают. Ты тут долбоеб.
107 481469
>>1468
Сильно дохуя для РКН. Это ведь означает, что перегрузка 2,5.
108 481470
>>1416
Разве сложно запилить герметичную капсулу с микробами и поместить ее на Марс, чтоб наблюдать за ростом бактерий и делать шаги в сторону терраформирования? Я понимаю Луна уебищна и непригодна, но Марс же это невозделанная целина! Вместо этого какие-то вскукареки на тему "Хмм, может быть, когда нибудь найдем марсианские микробы, а может и нет, лол."
110 481472
>>1470
И что тебе даст герметичная капсула? Ну гравитация 0,4 и все, нихуя интересного.
DiagramofSaturnVLaunchVehicle.jpg1,1 Мб, 3000x2338
111 481483
>>1471
Стартовая масса Saturn V с Apollo около 2950 т, тяга пяти F-1 у земли 33,8 МН (от полёта к полёту были отличия). Ускорение 11,8 м/с2 на старте никак не получается. Очевидно, что это график перегрузки, но в США с обозначениями всё запутано.
112 481488
>>1483
Ебучие немытые варвары со своими футами и фунтами. Еще бы дюжинами считали, блять.
image.png6 Кб, 506x154
113 481490
>>1483
мудила блядь
~3,4 разделить на 2,95 будет как раз 1,15
114 481491
>>1472
Давление и уровень радиации же. Потом капсулу забрать и посмотреть результат длительного влияния ультрафиолета на земные организмы. Профит же.
115 481492
>>1491

>Герметичная


>Давление


Выбери одно.
116 481493
>>1491

>Давление


Ммм давление в герметичной капсуле

>уровень радиации


Проще воспроизвести

>Потом капсулу забрать


Для возврата с Марса есть более интересные эксперименты, не говоря уже о грунте

>посмотреть результат длительного влияния ультрафиолета на земные организмы


аналогично можно воспоизвести
117 481494
>>1490
Ещё сила тяжести действует, блджад.
Ты мало того, что невежда, так ещё и агрессивно отстаиваешь это.
118 481497
>>1494
Ииии? Ты что вообще доказать хочешь, что датчик g на ракете показывает меньше единицы или что?
119 481498
120 481500
>>1498
Ты хочешь сказать есть такое понятие как "ускорение за вычетом гравитационного влияния земли"?
121 481502
>>1500
Не понимаю, к чему этот вопрос. Но вообще такое часто рассматривается. Разумеется, это ускорения, которые вызывали бы некие силы, действуй только они без силы тяжести - да вот хотя бы в определении перегрузки так. Ну и второй закон Ньютона позволяет рассматривать ускорение не только как отношение равнодействующей силы к массе, но и как сумму ускорений, которые вызывали бы силы по отдельности.
122 481503
>>1502
Тогда какой смысл писать про 0,2-0,6ж если эти цифры никто не считает и человек на борту ракеты будет чувствовать 1,2-1,6ж?
123 481505
>>1503
Вопрос-то был про ускорение.
124 481506
>>1505
В контексте здоровья же. А человеку похуй чувствует ли он чистые 1,5ж или 1ж от земли + 0,5 от ракеты
И да, ускорение никто не измеряет за вычетом гравитации.
125 481513
>>1506

>чистые 1,5ж или 1ж от земли + 0,5 от ракеты


Ну приехали. На старте сила тяжести действует вниз. Так что если двигатели вызывают ускорение меньше g, никто никуда не полетит.

>ускорение никто не измеряет за вычетом гравитации


В таком случае на старте и получаются для большинства РКН 0,2...0,6 g - вместе с действием силы тяжести, дающей 1 g в противоположном направлении.
А в инженерных расчётах часто рассматриваются действия только некоторых определённых сил и соответствующим образом считаются ускорения.

>В контексте здоровья же


Ну тут да, важна перегрузка, а не собственно ускорение. Но они же всё равно конкретным образом связаны.
126 481522
>>1492
>>1493
Справедливости ради, герметичность вовсе не означает отвязки от внешнего давления. Вполне можно сделать какую-нибудь герметичную бочку с уравниванием давления через эластичную мембрану.
127 481529
>>1522
Ну да, но нахуя просто это все на Марс везти? Будет только марсианская сила тяжести (проще уж раскрутить центрифугу на орбите) и марсианский уровень радиации (воспроизводится искусственно где угодно). По условию герметичности, марсианская атмосфера и почва не допускаются. Дорого и технически сложно, а смысла - примерно ноль.
128 481555
Какое минимальное и среднее расстояние между звездами в шаровых звёздных скоплениях наподобие M75?
129 481556
>>1555
И как они держатся вместе? Вращаются вокруг общего центра масс?
130 481561
>>1555
Минимальное хоть 0 у контактных двойных, среднее порядка 1.5 световых лет, если брать данные М75 в 400000 звезд и радиус 67 световых лет. Но плотность неравномерно распределена, в середине концентрация гораздо выше, вплоть до расстояний <0,1 светового года.

>>1556
А как еще-то?
131 481563
>>1561
Я имел в виду, что в центре скопления нет ЧД или типа того.
132 481564
>>1563
По-разному бывает, у некоторых есть, у некоторых нет.
133 481566
>>1563
Ты преувеличиваешь значение ЧД и "массивного центра" вообще. Если из центра нашей галактики убрать ЧД, она не развалится. В солнечной системе планеты действительно вращаются вокруг Солнца, потому что в Солнце 99% всей массы системы, а вот про Млечный Путь нельзя сказать, что он вращается вокруг супермассивной центральной дыры. По оценкам, масса Стрельца A* - 4-4,5 млн солнечных, а масса всей галактики - 480 млрд солнечных. То есть, в центральной ЧД собрано менее 0,001% общей массы.
134 481615
>>0719 (OP)
Спутники выше ГСО наблюдаемо движутся в обратную сторону?
135 481617
>>1615
Верно
136 481618
>>1615
Как Луна
137 481735
>>1566
Почему тогда в центре большинства галактик есть сверхмассивные черные дыры? Разве центробежные силы не должны выталкивать их наоборот из галактик?
138 481756
>>1735
Потому что они в центре, где дохуя вещества. Вещество "слипается" - получается ЧД. Короче, ЧД в центре - это следствие, а не причина образования галактик.

Центробежные силы - это вообще костыль, псевдосила. Нет такой хуйни по-хорошему. Чтобы ЧД из галактики вылетела, она должна иметь нехуйскую скорость. А ее нет. Поэтому и не вылетает.
139 481761
>>1756
Это ты сам придумал или есть исследования и расчеты на эту тему ?
140 481770
>>1761
Что сам придумал? Ну вот тебе из педивикии:

> Центробежная сила[1] — составляющая фиктивных сил инерции, которую вводят при переходе из инерциальной системы отсчёта в соответствующим образом вращающуюся неинерциальную. Это позволяет в полученной неинерциальной системе отсчёта продолжать применять законы Ньютона для расчёта ускорения тел через баланс сил.



Рассмотри тело, летящее вокруг другого по орбите. На него действует ровно одна сила: гравитация, которая направлена к центральному телу. Никаких центробежных сил там нет. И чтобы улететь, телу нужна скорость, потому что тогда гравитационно-обусловленное ускорение не успеет его "завернуть".
141 481771
>>1770
Я другой аноним, я про:

"Потому что они в центре, где дохуя вещества. Вещество "слипается" - получается ЧД. Короче, ЧД в центре - это следствие, а не причина образования галактик."
142 481772
>>1771
Ну я не цитирую какие-то исследования, можно сказать, что сам придумал.

Вот мои рассуждения: масса центральной ЧД (по крайней мере в нашей галактике) пренебрежимо мала по сравнению с массой всего вещества. Более того, такие ЧД только растут (их хоккинговсое испарение гораздо медленнее поглощения мимовещества), так что при образовании галактики они были еще меньше, вероятно в разы, если не на порядки. Размер галактики огромен, поэтому подавляещему большинству вещества вообще похуй, что там масса в центре из себя представляет: на расстоянии гравитационное воздействие ЧД ничем не отличается от воздействия такой же массы в виде обычной звезды или системы звезд. Поэтому, как мне видится, ЧД никак не могла стать причиной образования галактики. Более плотное, чем обычно, скопление вещества - да.
143 481775
>>1772
Логика есть конечно.
144 481797
Чому фоткали месье 87, а не ченить поближе?
Хайрез фоточке чернодыры когда?
Хайрез нейтронной звезды когда?
145 481800
Можно ли на Луне использовать 5-10 метровой высоты ходули для быстрого передвижения? Сами посудите - гравитация низкая, усилия для передвижения почти не требуются, можно как страус ебашить на ходулях перешагивая через мелкие кратеры, как босс. Вон они телескопические какие есть, если надо увеличиваешь высоту на 10 метров и вообще можешь увеличить скорость передвижения без всяких колёс. Или есть какие-нибудь минусы?

Если скажате, что типа можно упасть, то я придумал систему амортизации типа щита, который имеет другие усы-антенны. Падаешь если, то выставляешь щит, выдвигаешь антенны и они съедают импульс от падения.
146 481801
>>1797
наиболее удобный вариант
это и есть хайрез
пока не слышал о планах
147 481802
>>1800
Огурцы на луне медленно елозют не столько из-за того, что ноги короткие, а из-за того, что скафандор дохуя жирный и чтоб случайно не наебнуться и не продырявить этот самый скафандор, ускорив знакомство с Аллахом. А на ходулях риск наебнуться и познакомиться с Вишну как раз усиливается многократно.
Щит какпитана Америки надо успеть среагировать, а когда ты в кучном скафандуре и без того укачанный легкой гравитацией, да еще и на ходулях как бухой балансируешь - щит скорее увеличит шансы повстречать Яхве, чем отложит.
Пока что лучше тачки для покатушек не придумали, садись и катись.
kenleung0026620px.jpg151 Кб, 620x902
148 481803
>>1802
Кароче щиты на груди и на спине нужно встроенные делать, как у черепах-ниндзя. И чтобы если наклонишься опасно - из тебя шипы резко выскакивали бы. Под астронавтом на ходулях лучше не ходить, он может на тебя упасть и продырявить своими штангами.
149 481804
>>1801

>наиболее удобный вариант


Она жеж в ебических ебенях, поближе ниче не было что ли?

>это и есть хайрез


Мутное говно уровня фотки блутона до горизонтов это хайрез?

>пока не слышал о планах


Хуево.

Вот еще важный тупой вопрос:
какого хуя в книжках по астрогномии и научпопе до Интерстелора показывали черную дыру как какую-то хуйню типа пик 1 безо всякого линзирования, ведь линзирование было известно еще дохуя давно, а первое изображение смоделированное на ибм в 1979 году уже вовсю показывало и неравномерный свет от креационного диска и искажения вовсю понятны и видны были...
Почему только Нолан нормально изобразил ЧД и только тогда зачесались реальный вид показывать!? А!? Суки, мудачье.
150 481805
Плюс, хули вы будете делать, если лифт сломается? ммм???? мммммммм? В БФР чтобы залезть точно нужна либо стремянка запасная, либо веревочная лестница, либо ходули. Ну и другие ландеры делают огромными сейчас - descent stage на жопе, ascent stage и лишь потом капсула. Всё это на высоте 10 метров как раз может быть
giphy (1).gif79 Кб, 750x432
151 481806
>>1805
Решать проблемы будем по мере появления.
БФР еще три раза поменяется до реализации, релакс.
152 481807
>>1802

>Щит какпитана Америки надо успеть среагировать


Да там долго же падаешь - это как слоу мо падение, можно успеть. Дело в мастерстве. Я легко могу представить быстрого атлета космонавта на Луне, в лёгком скафандре, который выдвигает ходули на 10 метров, подпрыгивает, подтягивает ноги под себя, балансирует на огромной высоте, как баллерина, крутится на носке, бегает, делает кувырки. Я думаю, что дело в сноровке, которую можно приобрести. А еще можно комбинировать систему маневровых двигателей в скафандре в добавление к ходулям - если ускоряешься слишком быстро, то система автоматически снижает падение за счёт маневриков-движков.
153 481808
>>1807

>еще можно комбинировать систему маневровых двигателей в скафандре в добавление к ходулям


У нас и так скафандеры ебически тяжелые, не можем нормально облегчить не рискуя разошедшимся швом на жопе и последывающим разматыванием кишок, а ты еще утяжелять хочешь взрывоопасным реактивным двигателем(-ями).
Ты прямо совсем огурцов не ценишь, уж больно хочешь их с Сатаной познакомить.
154 481809
Можно ли на спутнике газового гиганта лампово сычевать, или поджарит к хуям обязательной радиацией?
Какие подводные профиты, какие камни?
155 481810
>>1808
Мне надо было уточнить, что я представляю это не через 10 лет конечно, но через 100 лет могу представить туристический закончик на Луне, огороженный, очищенный от острых камней, куда туристов будут пускать побегать по поверхности на ходулях, пошвырять песок, поиграть в гольф, футбол, боулинг, другие игры с мячом и шайбой, чтобы оценить как низкая гравитация влияет. Всё это если делать под контролем космонавтов-супервайзоров, то можно избежать риска и люди смогут там получить позитивные ощущения.
156 481812
>>1810
Нафига это на поверхности делать? А ну как вспышку сдетектют - придется загонять огурцов под реголит, побегут в панике, повбивают друг друга, закроют лунную программу, средневековье, гроб гроб кладбище Илон.
Если хочется анальной акробатики достаточно спиртзал выкопать и обмазать матами, пущай полетают без риска выпадания кишечника, поджарки на слонце и проткнутых гениталиями шлемов.
А опасную работенку в будущем пусть роботы от Бостон Дайнемикс на пару с инвалидом-Федей выполняют.
157 481813
>>1809
Смотря на каком спутнике, какого газового гиганта и какую радиацию ты боишься. Они все разные. Ио, Европа - смэрть на поверхности от излучения из радиационных поясов Юпитера. Ганимед не совсем смерть, в зависимости от того где ты и когда - у полюса или экватора. Каллисто нет. Но у обоих последних есть риск схлопотать риск от галактической радиации дозу со временем, или смэрть во время коронального выброса от Солнца. У Сатурна магнитное поле слабее, там легче везде почти. Уран и Нептун не знаю.
158 481814
>>1813
Не обязательно наши безжизненные сраные камни вокруг зловещих сатппррнов и пидеров, есть же дохуя экзопланет.
Рассматривают ли спутники газонюхов, может они вообще более приоритетные чем обычные соло-планеты? Типа магнитное поле из-за активной тектоники пижже, или еще какие профиты.
1466361342790.png386 Кб, 500x600
159 481816
>>1812
Чувак, поверхность это реальность, это бытие. Я уверен, что люди хотят в космос попасть не для того чтобы вечно зырить в окно, они хотят РЕАЛЬНОСТЬ почувствовать, что-то непередаваемое опосредованно через монитор или VR-headset. Поверхность Луны это самое то, если не вышел на поверхность, всё время на лунной базе был, в окно смотрел, то в будущем могут и засомневаться "а реально ли всё это, или же я где-то под Землей и зырю в мониторы?". В будущем идея 13-го этажа из фильма будет сильно людям нервы грызть, поэтому нужен непосредственный контакт с реальностью, как можно меньше фильтров, окон, линз, экранов. Мне кажется деньги будут платить именно за это. А то ведь можно будет на Луну и без риска слетать из дома, скачав какое-нибудь приложение типа SpaceEngine 9000 и побегать где хочешь в виртуальном шлеме без риска, платы, но и без ощущения.
Так что создать загон на открытом вакууме для туристов это самое то. Можно над ними пленку натянуть(нежелательно), или магнитное поле включить локальное, или делать ночью, чтобы на той стороне Луны, которая не освещена и не получала бы радиацию на случай выброса.
160 481818
>>1814
Для экзоспутников экзопланет всё также - зависит есть ли у них своё магнитное поле, глобальное оно или нет, какая у них орбита, не проходят ли они через радиационные пояса, как сильно магнитное поле газовика защищает их, как сильно активна звезда и много других там наверное переменных, если они подходят, то можно сычевать, но переменных оче много. Пока что вроде как одну экзолуну нашли, но это не точно пока что.
161 481863
Нашел телескоп PARALUX NEWTON 114/900 за 100 долларов в магазине, в чем подвох? Почему так дешево? Если не найду подвох - покупаю, блэт.
162 481883
>>1804
Не знаю о чем ты. В Spore 2007-го кажется года выпуска были вполне себе линзирующие. И во многих других космосимах, уверен, тоже.
163 481917
>>1863
Иди в телескопортед >>429065 (OP)
164 481931
>>1043
Бамп вопросу. Почему нельзя мотать круги с почти световой скоростью в сантиметре от горизонта событий (считаем, что вокруг дыры больше ничего нет, столкновения нам не грозят)?
165 481945
>>1043
>>1931
Потому, что метрика пространства люто распидорашена. У невращающейся дыры самое ближнее где в теории можно орбитировать это фотонная сфера в 2.5 радиусах шварцнеггера, только фотоны гипотетически могут на скорости света кругаля наворачивать, но они этого, естественно, не делают, ведь стоит твоей мамке повернуться на бок - гравитационные волны сместят хрупкий баланс орбиты фотонов и они либо ебанутся на чорную дырень (не мамки, а астрономический объект), либо свинтят в ебеня.
Дохуища видео было и есть на эту тему же, и статьи простные, все наглядно и понятно, всяко лучше, чем читать неадекватного питурда на двоще.
5a88309ac2955161a3fddcad (1).png17 Кб, 974x493
166 481950
почему луна влияет на воду так, как слева, а не так, как справа?
почему с воздухом, земной корой и мантией не также?
167 481953
>>1950
Потому, что ближайшую воду к луне притягивает луна, а потом в свою очередь... ближайшую Землю к луне притягивает луна, оставляя дальнюю воду в покое.
И да, с воздухом, корой, мантией, небом и Аллахом то же самое, просто вода наиболее заметна невооруженным взглядом.
168 481957
>>1945
То есть в сантиметре можно быстренько пролететь разок, но не стать на орбиту, так?
169 481966
>>1950

>почему с воздухом, земной корой и мантией не также?


Вощета, все с ними так же.
170 481967
>>1957
Вроде можно мимопролететь, да, только у тебя до подлета еще надо иметь дохуя кинетической энергии, чтобы назад отпустило, а то при близкопролетании завернет траекторию, как я помню.
С другой стороны, я не пробовал, так что можешь дождаться более адекватного анона.
171 481976
Как звучит особо мощный взрыв, например, метеорита или сверхмощного ядренбатона в очень большом далеке? Поблизости это ХДЫШЬ, СТЕКЛА ВЫБИТЫ, ПЕРЕПОНКИ ВЫДАВЛЕНЫ, ГРОБ ГРОБ КЛАДБИЩЕ ВЗРЫВ.
А мощный но очень далеко как звучать будет? Оно же подрассеется с расстоянием.
Знакомый мужик рассказывал, что в Мурманске был когда царь-батон жахнул, но звук не описал. Любопытно, как это звучало в разных частях, учитывая, что взрыволна по слухам три раза вокруг земли крутанулась.
Кстати, в противоположной точки земли от какого-нибудь чиксулубского уровня бабаха что будет?
172 481993
>>1950
С водой и воздухом еще отличие в том, что они не просто растягиваются, а еще и текут.

Приливные силы действуют на все предметы на Земле, даже твердые камни испытывают приливы, но эффект от этого крошечный, невооруженным глазом его не разглядеть. Только воздух в атмосфере и вода в океанах могут свободно обтекать всю планету и накапливаться в приливных буграх.
173 481995
>>1993
Планеты тоже жидкие.
174 481996
Анон, если где-нибудь сайт, где четко показано положение Земли, Марса, Венеры относительно друг друга в системе координат Солнца? Ну типа Солнце в центре, Земля здесь, там Марс здесь ближе всего сейчас и всё такое?
175 481997
>>1996
https://www.solarsystemscope.com
А вообще - практический любая программа-планетарий.
По идее даже Вселенная Песочница и Космический Двигатель могут, но я не уверен.
176 482009
>>1995
«Жидкие» они очень условно и разве что на геологических масштабах времени, на масштабах дней и недель «жидкая» мантия ничуть не мягче камня.
177 482011
>>2009
ПРЕДЕЛ




К счастью, жидкость планет мы не наблюдаем в нашей системе, ибо она давно устаканилась, но если пролетит какая Нибиру или Меланхолия Ларса Фон Триггера мимо планеты - распидорасит как гигантскую каплю.

Даже комету Шумахера-Левита распидорасило, хотя она сильно меньше планет и не стала круглой из-за гравитации, а все приливные силы.
2020 Mars.jpg322 Кб, 1165x743
178 482012
>>1997
Спасибо. Она немного громоздкая, но для демонстрации окна пуска миссий на Марс в 2020 подходит.
179 482013
>>2011
Пиздец, столько бреда, что даже поправлять нет смысла.
180 482014
>>2012

>громоздкая


Ну а что ты от веба хотел?
181 482015
>>2013
Поправлять нет смысла потому, что все верно.
Предел Роша - факт.
Планету в пределе Роша распидорасит как гигантскую каплю - факт.
182 482097
>>1804

>Вот еще важный тупой вопрос:


>какого хуя в книжках по астрогномии и научпопе до Интерстелора показывали черную дыру как какую-то хуйню типа пик 1 безо всякого линзирования, ведь линзирование было известно еще дохуя давно, а первое изображение смоделированное на ибм в 1979 году уже вовсю показывало и неравномерный свет от креационного диска и искажения вовсю понятны и видны были...


>Почему только Нолан нормально изобразил ЧД и только тогда зачесались реальный вид показывать!?


Бамп вопросу.
183 482125
>>2097
Показывали то, как есть (физическую структуру), а не то, как человек бы видел. Когда тебе в атласе карту Земли рисуют, ты же не удивляешься, а хуле на карте облаков нет, если из космоса их видно.

Так и здесь, с чего бы на схематическом изображении должны быть эффекты, зависящие от наблюдателя?
184 482128
>>2097
Пока эти искажения не зафорсили в интерстелларе хуйдожники не обращали на это внимания, считая очередной физической абстракцией. Ты бы ещё спросил почему аккреацилнный диск красным рисуют.
image.jpeg274 Кб, 600x420
185 482143
:(
186 482232
>>1804

>поближе ниче не было что ли?


Просто она находится строго над нами и никаких помех при наблюдении(фотографировании) не было. Представь два перпендикулярных диска, вот мы тот что снизу.
187 482246
>>2232
Плюс еще тот факт, что у М87 здоровенная дырень, по угловым размерам не уступающая СМЧД Млечного Пути, и без заслоняющего ее галактического говна.
188 482264
>>2232
Андромеда же в 20 раз ближе.
Хотя СМЧД в 50 раз меньше. И галактика под углом.
Ну и ну.
Другие дыры фоткать будут?
189 482265
>>2246

>по угловым размерам не уступающая СМЧД Млечного Пути, и без заслоняющего ее галактического говна.


Так говно непроницаемо для видимого, в микроволнах там все видно, стрельца уже давно смотрят, просто видят лишь звезды ходящие кругами под сабатон вокруг.
190 482266
Как чёрная дура образуется изнутри? Пусть злые алиены равноудалённо разместили вокруг тебя кучу нейтронных звёзд с максимально равномерным распределением массы. Они начнут сближаться к центру (с тобой в нём) и вскоре расстояние будет меньше радиуса шварцнегера, т.е. для внешнего наблюдателя чд уже сформировалась, а что будет видеть внутренний и как образуется сингулярность прямо в месте, где он стоит? Его же к звёздам вокруг тянуть будет.
image.png181 Кб, 565x541
191 482268
>>2266
Ты задал охуенный вопрос. Тоже хочу ответов на него увидеть.
192 482288
>>2266
Приливными силами распидорасит до слияния. Если у тебя неубиваемый и неразрываемый дыронавт, то в какой-то момент "видеть" перестанет быть физиологическим процессом. Впрочем как и вообще чувствовать. Физиологические процессы действует в плоскости бытовой, не экстремальной физики. Что до формирования ядра сингулярности, то это дело воображения. Наука нынче не рассматривает все эти умозрительные эксперименты. Но здравый смысл подсказывает, что это будет не внезапное локальное возникновение точки пространства содержащей всю массу ЧД, а вполне подверженный принципам небесной механики с временными рамками процесс поглощения вихрей материи.
193 482290
>>2288
Я думаю он о другом - о разнице между временем схлопывания у внешнего наблюдателя (у которого время течет обычно) и внутреннего, у которого время течет все быстрее.

>>2266
У тебя просто версия парадокса близнецов, т.е. и не парадокса вовсе. Просто учти, что чтобы узнать точное время снаружи, находясь внутри, надо как-то провзаимодействовать. Взаимодействие не передается быстрее с, и тоже подвержено гравитационному влиянию. Если вся масса уже схлопнулась под радиус шварценфельда - взаимодействия никакого не будет, если вот-вот схлопнется - сигнал тоже будет замедляться/ускоряться.

Как будет выглядеть формирование изнутри - время замедляется, снаружи ускоряется, звездочки моргают всё быстрее, визуально картинка выворачивается наизнанку линзой, в центре (куда ни посмотри) образуется черный пузырь и заполоняет всё поле зрения. А что будет дальше - никто не знает, на этом моменте известный физон ломается к чертям собачьим.
194 482319
>>2288

>Что до формирования ядра сингулярности, то это дело воображения. Наука нынче не рассматривает все эти умозрительные эксперименты.


Вполне рассматривает. Голая сингулярность, кольцевая во вращающихся етц.
Это математический конструкт.
195 482333
>>2265
«Все видно» может означать как 100% проницаемости, так, и скажем, затухание на два-три порядка. Конкретных цифр я не видел.
196 482389
>>1976
Высокие частоты с расстоянием теряются. Условно говоря, из ссс превращается в шшш, потом в ввв, потом в ууу. Точное направление звука с расстоянием определить всё сложнее, потому что звук, как ты подметил, рассеивается, и получаются многократные переотражения, роль которых теряется с занижением частоты и со сменой среды-носителя.

На обратную сторону Земли доходит только глубокий инфразвук, в основном по коре. Он даже на низкую орбиту доходит, где механическое взаимодействие молекул уже практически невозможно, но надо чтобы он был очень инфра- и очень сильный: https://agupubs.onlinelibrary.wiley.com/doi/full/10.1002/grl.50205
197 482390
>>2389
То есть взрыв очень мощной бомбы очень далеко прозвучал бы как раз как "Вжжжжж", без резкого буха, так ведь?
198 482391
>>2390
Скорее более плавное уУУУУУУуууу, как в метро или при землетрясении. Все от расстояния зависит. Вот эти три раза вокруг земного шара могут дохуя чего означать - типов волн в сейсмологии дохуя, да и на излете ухом не услышать было бы, даже приложив к тектонической плите. (фигурально)
199 482393
Какая примерно терминальная скорость у капсулы с людьми на уровне моря? без учета парашютов
200 482394
>>2393
Без парашютов примерно 100-120 м/с.
201 482411
посмотрел только что
https://www.youtube.com/watch?v=uD4izuDMUQA

что думает анон по поводу того что будет когда все все все звезды сьедят свое топливо и исчезнут? что будет когда исчезнет последняя черная дыра во вселенной?
202 482412
>>2411
нихуя не будет
тепловая смэрт же
203 482494
>>2391
Даже чиксулубский уровень который выпилил дохуя биоты?
Если у тебя есть возможность времятуризма, то что ты увидишь в епропке когда импактор хуйнул в сракотан ЮКОТАН, Ю, не сра.
204 482495
>>2411
Вроде дохуя виде вполне доступно показывают о пиздеце. Энтропия, все дела.
205 482718
Если бы наша система была в межгалактическом пространстве чтоб ни одной звезды, кроме солнца, в радиусе 1кк св.л, в каком году мы бы открыли, что существуют другие звёзды?
206 482727
>>2718
Если бы были другие планеты,то к середине 20 века появился бы аналог радиотелескопа, с их помощью обнаружили бы. Если планет нет, или атмосфера непрозрачая то скорее всего и не обнаружили бы вообще, если только случайно.
207 482728
qwe
208 482757
>>2718
какой же тупой вопрос блять....
если бы звезд не было видно глазом, то никто бы не строил телескопы чтоб посмотреть вверх. то есть рандомно кто-то бы какую-то залупу бы направил и может что-то увидел, но это тупо рандом, это могло случиться и в 2000 году и в 3000
209 482761
Видел видео обьясняющее что земля каждые 2 с небольшим года поворачивается, это делается чтобы орбита была круговой. Нихуя не могу понять что это значит и видео найти не могу.
210 482762
Если в Земле прокопать охуенно глубокую дырку, на какой глубине воздух или какие-то его компоненты начнут сжижаться из-за давления? Считаем, что из-за хитрой магии края дырки не обваливаются, мантия не плавит, вода не заливает.
211 482775
>>2762
В Земле глубины не хватит.
212 482780
>>2775
А давление газиков разве не экспоненциально растет? Воды почти линейно - это я знаю, - но ведь вода практически несжимаема.
213 482782
>>2757
>>2757

> 482757208


ну это понятно. нормальный вопрос
214 482786
>>2757
Туманность Андромеды от нас в 2,5 млн световых лет, а невооруженным глазом вполне видна. Так что совсем пустым небо не было бы. Опять же, если есть планеты, астрономия могла бы развиться чисто как наблюдение за ними.

Даже без этого есть реликтовое излучение. Его обнаружили бы вскоре после изобретения радио для местных нужд, после чего заинтересовались бы, откуда эта хрень берется и нашли бы всякие квазары. Отсюда всплеск интереса к астрономии, обнаружение прочих (неактивных) галактик, сопоставление со своей звездой. Наверняка бы просекли, что галактики - скопления звезд, никаких 3000 годов тут надо.
215 482787
>>2786

>тут надо


тут не надо
Слоуфикс.
216 482825
https://www.youtube.com/watch?v=bhw6ZBC6gmY

Обратите внимание на 0:30 - 0:40 секунды.

Что случилось с небом? Почему оно почернело когда ракета дошла до облаков и потом вернулось в нормальный вид?
217 482826
>>2825
Просто тупая камера светочувствительность крутанула за каким-то хером.
218 482827
>>2825
от гептила такие эффекты бывают
посветлело потому что выхлоп пошел в сторону
219 482828
>>2825
В кадре появился факел ракетного двигателя, который сбил с толку автоматику видеокамеры, чуть позже она отрегулировалась. Это на видео кажется что ну пламя и пламя, а вживую когда пуск РКН наблюдаешь, оно пиздец яркое и слепит не намного хуже электросварки или солнца.
220 482833
>>2494

>Если у тебя есть возможность времятуризма, то что ты увидишь в епропке когда импактор хуйнул в сракотан ЮКОТАН


Если и почувствуешь что-либо, то будет как при землетрясении, скорее всего. Лучше поближе брать. (хотя европка к Юкотану поближе была 65 млн лет назад)
1547400301777.png77 Кб, 240x240
221 482834
>>2827
Ломающие новости, облако гептила заслонило небо и привело к новому ледниковому периоду!
222 482840
>>2825
Сингулярность стала поглощать фотоны света.
223 482841
>>2840
Протоны света, ты хотел сказать?
224 482844
>>2833
Абажжи, а как тогда все умерли нахуй, если это всего лишь землетрясение?
225 482846
>>2844
Умерли сразу те, кто недалеко были (хотя "недалеко" - это сотни километров, если не тысячи). Остальные крупные дяди вымерли из-за изменения климата ("ядерная" зима, вероятно глобальные пожары) и вызванного им разрушения пищевых цепочек. Банально жрать стало нечего.
226 482847
>>2846
Ещё от цунами, наверно.
227 482942
Марс с Титаном более менее одинакового размера, при этом у Марса атмосферы - процент от земного, а у титана ажно в 1,5 раза больше. Почем так? Где корреляция?
Могут ли быть какЗемли - без атмосферы или с атмосферой тоще лунной или Плутоины с атмосферами как на Земле?
228 482943
>>2942
У марса СДУЛО, у титана НАДУЛО.
Марс тут один, поближе к солнцу, без спутника и магнитосферы соснул тунца, в то время как титан подсасывал от ближайшего газонюха в относительном далеке от губительного для атсфомеры солнца.
229 482944
>>2943
Земля Марса на порядок ближе к Солнцу. Новейшие исследования показали, что атмосфера Земли вытягивается аж за пределы лунной орбиты. Почему тогда всю эту легко взвешенную атмосферную пыль не сдувает солнечным ветром?
230 482945
>>2944
МАГНИТОСФЕРА.
А еще благодаря массе живая тектоника и пополнение атмосферы попукиванием в нее.
Венера вообще ебанулась на отличненько и больше 100 атмосфер набздела, словила парниковый эффект и индуцированную магнитосферу.
231 482946
>>2942
Титан далеко от Солнца и холодный, Марс близко и относительно теплый, молекулы быстрее улетают в космос.

И даже при всем при этом Титан с момента формирования Солнечной системы потерял охуительнейшее количество газа, суммарной массой в несколько десятков земных атмосфер. Просто изначально у него атмосферы гораздо больше было.

>Могут ли быть какЗемли - без атмосферы или с атмосферой тоще лунной


Могут.

>или Плутоины с атмосферами как на Земле?


Как на Земле, с той же температурой — нет, сдует за считанные миллионы лет, но гораздо более холодные — могут.
232 482955
>>2942
Ты не учитываешь что атмосфера может образовываться в результате вулканизма или ещё какого процесса, и банально что её изначально могло быть очень дохуя. Та же Венера тоже меньше Земли немного, но газа в атмосфере во много раз больше, хватит на несколько Марсов и Земель и ещё останется столько же.
233 482956
>>2955
Копание мохола поможет напердеть в атмосферу марса, или надо восемьсот тысяч комет в атмосфере распылить?
234 482959
Если в космосе пукнуть облачком кислорода/воздуха из какой-нибудь бочки, то можно ли будет в этом облачке находиться без скафандра?
235 482960
>>2946
почему тогда на ганимеде, который почти во всё идентичен титану, вообще нет атмосферы?
236 482964
>>2959
Не можно, очень быстро рассеется, потому что газ стремится занять весь предоставленный объем.
237 482966
>>2964
а вот и нихуя. в космосе газ считай жидкость. и будет он липнуть к эпицентру. если в центре - человек, газ окружит его и прилипнет. чем больше газа, тем больше налипнет. если вообще много, человек захлебнётся водой, появившейся в конденсате на дне такого с позволения сказать планетоида, и вся вода будет скапливаться и стекать туда: человек не сможет из неё выбраться, ибо вся вода будет течь на него
238 482967
>>2966
Броуновское движение, мазафака, ты слышал про него?
Если ты газики не охладишь до чуть ли не конденсатора бозона-эзейнштейна, то у молекул будет достаточно энергии чтобы невозмбранно разлететься нахуй в ебеня.
Либо охлади к хуям чтобы кинетическая энергия не превышала гравитационную, либо делай планеты.
239 482968
>>2966

>в космосе газ считай жидкость


А инженеры то и не знают, скафандры какие-то там городят! Зочем, если анон с харкача сказал, что газ в космосе к человеку липнет, ну охуеть теперь.
240 482983
>>2955
Всякие магнитные поля вторичны, единственное что имеет значение это масса планеты и вулканизм. Зеленый кот считал, будь на Марсе магнитное поле как у Земли атмосфера оттуда улетала бы всего на несколько процентов медленнее чем ИРЛ.
241 483032
>>2983
Зеленый кто?
242 483042
Кот
243 483043
А-а-а.
244 483103
Сейчас будет несколько очень тупых вопросов от бумагомарателя. Часть не по космосу, но куда писать-то еще?

1. Предположим, есть вогнутый сзади абляционный щит. Предположим, что проблем с его удержанием самим "щитом" в ретроград проблем нет. Насколько реально вернуться с орбиты вне капсулы, лежа в таком? Какие температуры будут на внутренней поверхности? А плазма не залетит?

2. Возможно защищаться от космических лазорей, выпуская, например, облако аэрозоля?

3. Какие перегрузки может выдержать современная сложная электроника? Баллистические ракеты со своими системами могут в 100g, а еще больше можно?

4. Ядерный дрыгатель - по сути, реактор, который охлаждается любым говном и сразу сбрасывает его через сопло. А ведь термоядерные реакторы тоже дохуя греются. А с ними так можно?

5. Говно - вроде, в основном все же углерод. Углерод выдает больше энергии, чем требуется на синтез. А можно ли закидывать в термоядерный реактор говно?
245 483109
>>3103

>1. Предположим, есть вогнутый сзади абляционный щит. Предположим, что проблем с его удержанием самим "щитом" в ретроград проблем нет. Насколько реально вернуться с орбиты вне капсулы, лежа в таком? Какие температуры будут на внутренней поверхности? А плазма не залетит?


А уже были проекты, ЛОСЬ, например. https://en.wikipedia.org/wiki/MOOSE
Муханизм, понятное дело, спецом под огурца, нельзя свинтить с союза крышку и огнетушителем свести себя с орбиты успешно.

>2. Возможно защищаться от космических лазорей, выпуская, например, облако аэрозоля?


Разумеется. Можно даже не выпускать его специально, корпус за тебя может это делать. https://en.wikipedia.org/wiki/Ablation

>3. Какие перегрузки может выдержать современная сложная электроника? Баллистические ракеты со своими системами могут в 100g, а еще больше можно?


Можно и больше, но на деле ты столько перегрузок не получишь используя реалистичную ракету - самая шустрая 400g. https://en.wikipedia.org/wiki/Sprint_(missile)
Есть электроника для артиллерийских снарядов, выдерживает 15500 g, https://en.wikipedia.org/wiki/Orders_of_magnitude_(acceleration)

>4. Ядерный дрыгатель - по сути, реактор, который охлаждается любым говном и сразу сбрасывает его через сопло.


Необязательно, но примерно так. Непрямоточные, например, не напрямую через активную зону охлаждаются, а есть маняидея реактора который запитывает ионники.

>А ведь термоядерные реакторы тоже дохуя греются. А с ними так можно?


А с ними как раз прямоток звучит как оптимальная стратегия - вместо многих контуров теплообмена напрямую сбрасывать многомиллионоградусную плазму, хотя и непрямоток для большего эффективности сгорания рассматривается. Только в отличие от ядерника, который хоть на какой-то бумажке виднелся, до термояда нам как зимбабвийцам до марса.

>5. Говно - вроде, в основном все же углерод.


Хуй там был, это в основном вода, на 3/4. Может меньше, если запор, но не сильно. В случае поноса ты сам видел из чего стул состоит.

>Углерод выдает больше энергии, чем требуется на синтез. А можно ли закидывать в термоядерный реактор говно?


В термоядерный реактор можно закидывать вообще все. А чтобы выхлоп был - в теории все что раньше железа в таблице мендельсона. А на практике, блядь, даже литий сфьюзить на постоянке не могут.
Отвечая по сути вопроса - да, говно жечь можно.
Можно кал на солнце отправлять, оно будет облизываться и просить еще. А вот если чугуна налить на него, оно охуеет и киснуть будет. Но там нужно не одну планетарную систему на это расхуярить под ноль, так что невалидно.
245 483109
>>3103

>1. Предположим, есть вогнутый сзади абляционный щит. Предположим, что проблем с его удержанием самим "щитом" в ретроград проблем нет. Насколько реально вернуться с орбиты вне капсулы, лежа в таком? Какие температуры будут на внутренней поверхности? А плазма не залетит?


А уже были проекты, ЛОСЬ, например. https://en.wikipedia.org/wiki/MOOSE
Муханизм, понятное дело, спецом под огурца, нельзя свинтить с союза крышку и огнетушителем свести себя с орбиты успешно.

>2. Возможно защищаться от космических лазорей, выпуская, например, облако аэрозоля?


Разумеется. Можно даже не выпускать его специально, корпус за тебя может это делать. https://en.wikipedia.org/wiki/Ablation

>3. Какие перегрузки может выдержать современная сложная электроника? Баллистические ракеты со своими системами могут в 100g, а еще больше можно?


Можно и больше, но на деле ты столько перегрузок не получишь используя реалистичную ракету - самая шустрая 400g. https://en.wikipedia.org/wiki/Sprint_(missile)
Есть электроника для артиллерийских снарядов, выдерживает 15500 g, https://en.wikipedia.org/wiki/Orders_of_magnitude_(acceleration)

>4. Ядерный дрыгатель - по сути, реактор, который охлаждается любым говном и сразу сбрасывает его через сопло.


Необязательно, но примерно так. Непрямоточные, например, не напрямую через активную зону охлаждаются, а есть маняидея реактора который запитывает ионники.

>А ведь термоядерные реакторы тоже дохуя греются. А с ними так можно?


А с ними как раз прямоток звучит как оптимальная стратегия - вместо многих контуров теплообмена напрямую сбрасывать многомиллионоградусную плазму, хотя и непрямоток для большего эффективности сгорания рассматривается. Только в отличие от ядерника, который хоть на какой-то бумажке виднелся, до термояда нам как зимбабвийцам до марса.

>5. Говно - вроде, в основном все же углерод.


Хуй там был, это в основном вода, на 3/4. Может меньше, если запор, но не сильно. В случае поноса ты сам видел из чего стул состоит.

>Углерод выдает больше энергии, чем требуется на синтез. А можно ли закидывать в термоядерный реактор говно?


В термоядерный реактор можно закидывать вообще все. А чтобы выхлоп был - в теории все что раньше железа в таблице мендельсона. А на практике, блядь, даже литий сфьюзить на постоянке не могут.
Отвечая по сути вопроса - да, говно жечь можно.
Можно кал на солнце отправлять, оно будет облизываться и просить еще. А вот если чугуна налить на него, оно охуеет и киснуть будет. Но там нужно не одну планетарную систему на это расхуярить под ноль, так что невалидно.
246 483110
1. Всё зависит от баллистического коэффициента, но вообще турбулентная зона и зона отрыва очень горячие, так что доедешь все равно даже не с корочкой, а в виде уголька. Может быть, если взять что-то типа НТУ или PARACONE с хорошим баллистическим коэффициентом, можно получить низкотемпературное реентри, для которого хватит и кевлара, намазанного говном. Но только может быть.

2. Аэрозоль разлетится жи

3. Есть управляемые снаряды, которыми из артустановки стреляют. Там какая-то электроника точно стоит. В /wm/ сходи, там должны знать.

4. Можно и нужно, только никто не знает как
247 483111
248 483155
>>3109

>https://en.wikipedia.org/wiki/Orders_of_magnitude_(acceleration)


Ебать, стрекательные клетки медузы развивают 5,5 млн g! Охуеть, никакой космос не нужен.
249 483171
С точки зрения ОТО, гравитация - не сила, а искривление пространства. Значит ли это, кто Земля, мотыляющая по орбите вокруг Солнца, движется равномерно и прямолинейно, просто вот такие прямые херовые? Является ли система отсчета, связанная с Землей (только невращающаяся), инерциальной в ОТО?
250 483218
Почему все сверхновые 1 типа взрываются с почти одинаковой мощью? Ведь белые карлики разные бывают: один из преджелезных элементов и массой почти достаточной для схлопывания, другой из гелия с пюрешкой и лёгенький. Большую же часть энергии взрыва обеспечивает водород, перетекший с соседа, значит и взрывы разные должны быть...
251 483228
>>3155
На молекулярном уровне всё намного прочнее, чем в привычном нам мире. Какая-нибудь графеновая микроструктура вообще могла бы миллиарды g выдержать.
252 483233
>>3228
Нуклоны посмеиваются над молекулопроблемами.
Эй, электрон не потерял?
13461039600.jpg10 Кб, 253x246
253 483250
Если бы в нашей Солнечной системе был Горячий Юпитер на орбите Венеры, значит ли это, что мы могли бы проводить очень дешевые гравити-ассисты на достижение третьей космической скорости и вообще на передвижение внутри солнечки? Кому-то так везет там среди звезд на таких вот партнёров и они могут наверное осваивать космос быстрее и дешевле.
254 483258
>>3250
Чтобы лететь внутрь нужно тормозить, чтобы наружу - разгоняться. То есть, чтобы провести маневр у такого юпитера на орбите Венеры, нужно было бы сначала затормозить (ну или, точнее, вылететь с Земли соответствующим образом). Поэтому кажется сомнительным, что это было бы эффективнее, чем разгон у собственно Юпитера.
255 483259
Вопрос про ЧД. Я считал, что ЧД это типа нейтронной звезды, но с гораздо большим сжатием. Но недавно смотрел документалку BBC, где один ph.d заявил, что такое мнение абсолютно неправильно.
Материя в ЧД переходит в другое состояние и ее, как таковой, не существует.
Тогда один вопрос, почему ЧД измеряют в солнечных массах, если материи там уже нет?

И, да
Я не зеленый.
256 483263
>>3259
Сингулярность - тот еще кот Шредингера в макромире. Есть предположение, что ученый подразумевал исчезновение всякой размерности и следом неприменимости бытового представления о материи. Но масса у сингулярности самая "чтонинаесть" объективно реальная. Еще есть вариант, что ты не въехал в контекст сказанного, додумав его в рамках своих неполных представлений. Хочешь более конструктивной мысли - тащи ученого сюда прям за его слова.
257 483266
>>3259
Потому что гравитирующая масса ровно такая же, как у сколлапсировавшей материи.

>Материя в ЧД переходит в другое состояние и ее, как таковой, не существует.


Ну хуй знает. Ему, по идее, виднее, но вообще я не слышал о каких-то общепризнанных и полных теориях что именно находится в черной дыре. Без стыковки квантмеха и ОТО все это не более чем более-менее обоснованное гадание.
258 483275
>>3258
А зачем тогда используют гравитационные ассисты у Венеры на миссиях к Юпитеру?
259 483276
>>3275
Так я не говорю, что невыгодно снизить скорость, чтобы спуститься к более ближней к Солнцу планете и сделать у нее гравманевр. Не предполагаю, для полетов из СС выгоднее сам Юпитер, чем такая же масса на месте Венеры.
260 483277
>>3276

>Не предполагаю


Но предполагаю
Слоуфикс.
261 483281
>>3277
Не, ну смысл именно в МАССЕ же. Если поменять Юпитер и Венеру местами и попытаться сделать гравиманевр от Венеры, которая на орбите Юпитера, то результат будет хуже же.
1384340175447.png71 Кб, 619x550
262 483282
>>3218
Потому что углерод-кислородных белых карликов более 90%, и хорошо взрываются только они.

>Ведь белые карлики разные бывают: один из преджелезных элементов и массой почти достаточной для схлопывания


Преджелезные элементы это уже нейтронная звезда, белый карлик это максимум ну там неон (20 а.е.м) или марганец. Да и то при значительной доле неона и марганца никакой сверхновой не будет, карлик просто сожмется в нейтронную звезду без катастрофического взрыва.

>другой из гелия с пюрешкой и лёгенький


А этих не только крайне мало, так они еще и вообще не взрываются.

>Большую же часть энергии взрыва обеспечивает водород, перетекший с соседа, значит и взрывы разные должны быть...


Чиво блядь? Внешние оболочки звезды вообще взрываться не умеют, для этого там и близко нет нужных условий. Взрывается углерод в центре белого карлика, перетекающее с соседа вещество только создает достаточную температуру и давление для того, чтобы в ядре возобновился синтез и оно пиздануло.

В процессе взрыва углерод выгорает практически полностью, кислород частично, вклад остальных элементов во взрыв вообще минимален.
263 483289
>>3281
Ну да. Я думаю, что ситуация как сейчас лучше для полетов из СС, чем если бы Венера и Юпитер поменялись местами. Два юпитера было бы еще лучше, конечно (но не факт, что не было бы хуже в чем-то другом).
264 483294
>>3282

> Внешние оболочки звезды вообще взрываться не умеют, для этого там и близко нет нужных условий


Ну они начинают падать, давление увеличивается, температура тоже и взрыв углерода тоже подогревает.
15575778671512.webm2,9 Мб, webm,
1920x1080, 0:13
265 483299
Двач знает, что это за пёсель и что он делает в невесомости?
266 483300
>>3299
Это реклама. Собаку скорее всего покатали на вомит-комет и потом врезали на фоточку модуля Кибо.
https://www.youtube.com/watch?v=45tK7szJFx0
267 483301
>>3300
вот это оперативность, да ещё и дабл-дабл
спосеба, анон
268 483311
>>3294

>Ну они начинают падать, давление увеличивается, температура тоже



Достаточное давление и температура для взрыва создается только при быстром коллапсе внешних слоев массивных (от 10 солнечных масс) звезд, при медленном перетекании вещества на крошечный белый карлик там и близко нет ничего подобного.

Просто перестань нести хуйню и запомни, что в сверхновых типа Ia взрывается только углерод и кислород.
269 483320
>>3311

> при медленном перетекании вещества на крошечный белый карлик там и близко нет ничего подобного.


Да блин, я имею в виду с момента начала схлопывания бк в нз. Там как раз всё предельно быстро происходит. Раз уж углерод загорается, почему б и водороду не тоже, нехай на поверхности?

> и запомни


ок, святой отец
270 483323
>>3320

>Да блин, я имею в виду с момента начала схлопывания бк в нз.


Чиво блядь ×2.

Схлопывание белого карлика в нейтронную звезду и взрыв белого карлика как сверхновой это совершенно разные и взаимоисключающие процессы.

При взрыве сверхновой типа Ia от белого карлика не остается ничего, там выделяется столько энергии, что всю звезду расхуяривает в клочки. При нейтронизации и коллапсе в нейтронную звезду, соответственно, вспышки сверхновой не происходит, потому что кислород-неон-магнезиевое ядро так и не начинает термоядерный синтез и не взрывается.

Ни в одном из этих случаев водород и гелий так и не загораются, потому что их снаружи ничего не обжимает с достаточной силой, чтобы начать горение с положительной обратной связью.
271 483325
>>3300
Для этого совсем необязательно фрахтовать самолет, песик же не вращается в свободном виде, достаточно грамотных подвесов, хромакея и пятьсот голодных узкоглазых фотошоперов вручную жабящих эти подвесы.
Вона, кинцо у нас про невесомость вполне без вомиткомета снимают, а тут ради рекламы жизнью собаки рисковать?
1541012788155793833.jpg58 Кб, 700x466
272 483333
Лекция хоть и посвящена времени и путешествию в нем, но там больше все же про космические полеты. Я смотрел два раза, но так и не понял, что это за полет который позволяет вернуться в прошлое, это полет вокруг черной дыры?

https://www.youtube.com/watch?v=KMiLU31Tu7o
273 483334
>>3333

>1 час лекции с переводом


Ну нет, давай ты сам своими словами.
274 483341
>>3323

Я не имел в виду, что нз таки образуется, просто сократил формулировку, что преодолевается предел чандрасекара, бк начинает проваливаться в себя, но тут его дико пучит и разъёбывает. Хотя прочитал на енг вики, что эта модель недавно устарела.
275 483352
>>2942
Во-первых, на Титане атмосфера по-любому постоянно наполняется засчет вулканизма, вулканизм за счет приливных сил Сатурна
Во-вторых, Марс и Титан состоят из разных веществ, в этом отношении это принципиально разные небесные тела, поэтому странно их сравнивать
276 483362
>>3325
Зеро джи полёт не особо и дорого стоит, на самом деле. Но да, таки собачка ненатуральная и скорее всего подвешенная. Я думаю что с зеро джи было бы тупо труднее снять - в вомит комете псина бы охуела и не расслабилась, т.к. там короткие периоды секунд по 20, а потом плющит.
277 483364
>>3362
В вомит комете собачка могла бы оправдать название самолета, усложнив съемки. Так что просто достаточно было сделать как в кинематгорафе.
Кстати, мне кажется, что видел как на одном из полетов варминт-кермита собачка как раз таки прыснула содержимым желудка, после чего незамедлительно словила его и употребила обратно.
Или это у меня комбо-воспоминание как какого-то негра стошнило в вомбат-комбате и на ненависть всем шарики блевоты полетели, наложилось на многократные просмотры видео того как собака употребляет пищу повторно.
Хотя щас задумался и к своему величайшему стыду должен признать, что скорей всего я видел только фейковидео из теории большого взрыва, а настоящее видео блевотни в вломит-клоните я не видел.
Без всякого стыда, тем не менее, видео с собаками топящими за реюз на собственном примере.
278 483366
Помните, есть открытая компактная звёздная система с 7 каменистыми планетами, 3 из которых в обитаемой зоне. Если это не ТРАППИСТ, то я не помню этого названия. Но суть не в этом.
Представим некую планетарную систему с тоже каменными земле-марсо-венеро-подобными планетами. Их от 3 до 7 (пусть до 7) в системе, и нету там никаких Нептунов и Юпитеров.
Это была присказка, а вопрос ниже: насколько близко друг к другу должны располагаться орбиты планет в этой компактной системе, чтобы не разрушить друг друга, но при этом влиять друг на друга, раскачивая внутренние динамо механизмы и, при этом, стабильно поддерживать подвижность литосфер коры и воды на поверхности? А также что бы как Януса и Эпиметея была орбитальная рокировка, причём, всех планет в системе (чтобы рано или поздно самая ближняя планета оказалась самой дальней, и потом однажды вернулась бы на ближайшую орбиту - и так с каждой планетой).
Естественно, представляем систему без не только газ-гигантов, но и лун типа Луны и астероидных поясов со всякими Вестами и Церерами.
279 483374
>>3366
да как на трапписте примерно и нужно, ну может чуть пошире
дальше уже не будет взаимного влияния
280 483376
>>3374
А чё, там планеты орбитами меняются?
281 483377
>>3364
Ващето многим и полная невесомость для этого не нужна
https://www.youtube.com/watch?v=P2hf3JS6q0E
проигрываю с дегенеративных рязанских ебал и рожи Командора
282 483378
>>3377
Без невесомости блевотина вниз падает, а не парит, заманчиво блестя питательными шариками.
283 483379
>>3376
Нет, но согласно моделированию для стабильного существования системы нужен очень специфический орбитальный резонанс всех планет, что-то типа Юпитера и спутников они и по размерам похожи кстати
284 483381
>>3334

>Ну нет


Что, ты не можешь воспринимать такие объемы информации?
285 483388
>>3381
Не охота слушать что и так известно в поисках крупицы нужной кому-то другому информации, да еще и в переводе на русский язык, сука.
Если какой-то аспект оказался непонятен, его можно вычленить хотя бы.
286 483389
>>3381
>>3388
А еще очень важное, что когда кому-то что-то непонятно, ему лучше объяснить что непонятно своими словами, сам процесс объяснения если не позволит самому разобраться эффектом уточки, так поможет именно непонятный аспект пояснить в деталях вместо того, чтобы лекцию 101 устраивать.
287 483426
>>3171
Бамп вопrосу.
288 483431
>>3426
>>3171
Я не знаю...
Ты, похоже, тредом ошибся, мне кажется, что это вопрос не тупой.
[заплакал]
1389053049556.jpg51 Кб, 499x499
289 483435
>>3377

>Puking


>пукинг

290 483624
Короче, вода при нормальных условиях жидкая, с таким поверхностным натяжением, что можно на плавать на суднах, да и дерево просто так вообще не тонет.
А если бы мы взяли планету с жидкой водой в обитаемых условиях, но:
1) с большей массой (скажем в 2 земных) при нормальном давлении
2) с меньшей массой (в пределах от Луны до Марса) при нормальном давлении
3) с большей массой (скажем в 2 земных) при малом давлении
4) с меньшей массой (в пределах от Луны до Марса) при малом давлении
5) с большей массой (скажем в 2 земных) при большом давлении
6) с меньшей массой (в пределах от Луны до Марса) при большом давлении
где под нормальным давлением берём одну атмосферу - н.у. Земли.
Так вот. В каком из 6 типов условий предполагаемых планет плавать самому или на плоту окажется невозможно из-за законов физики?
291 483628
>>3624
Насколько я понимаю, везде ты сможешь плавать, потому что это зависит только от плотности воды, которая практически не меняется. При возрастании давления плотность чуть повышается (очень медленно), но это сделает плавание только проще.

При экстремально больших давлениях, возможно, начнет тонуть дерево, потому что оно сжимается куда лучше воды, а значит его плотность растет. Но при таких давлениях человеку один хуй кранты, тут уж не до плавания на плотах.
Снимок.PNG51 Кб, 424x402
292 483629
>>3624
Википедия, придурок малолетний, википедия.
293 483631
>>3629

>если бы мы взяли планету с жидкой водой


Почитал бы вопрос хоть, мудло.
294 483634
>>3628

>возможно, начнет тонуть дерево, потому что оно сжимается куда лучше воды, а значит его плотность растет


>начнет тонуть дерево


>потому что оно сжимается куда лучше воды


>а значит его плотность растет

295 483635
Что неверно в этом утверждении - «вероятность существования инопланетян равна 50% - либо существуют, либо нет»?
296 483637
>>3634
И смешно это потому, что?
А-а, ты просто не в курсе, что дерево легче воды из-за микропор в целлюлозных каналах, и про существование древесины тонущей в воде тоже не в курсе, да?
297 483638
>>3635
Все верно.
298 483639
>>3635
Это не вероятность, это... гм, для такой хуйни должно быть название.
Вероятность должна быть подсчитана и иметь значимую оценку. У такого же утверждения никакого смысла нет, это утверждение лишь два раза за текст говорит, что вариантов ответа в бинарном утверждении два.
299 483673
>>0719 (OP)

>Q: Что будет с человеком в вакууме без скафандра


>A: Он умрёт.


А от чего он умрет? Закипание воды в организме? Множественные повреждения от выходящих газиков? (Вряд ли задохнется, тренированые люди могут на несколько минут дыхание задерживать).

Что если оставить человеку шлем с подачей дыхательной смеси, герметично соединенный с кожей, но оставить все остальное тельце без защиты?
300 483702
>>3673
Именно задохнется.
Ты не можешь задержать дыхание в вакууме.
Через максимум 15 секунд полная потеря сознания.
А дальше обычная смерть как при удушье. Закипание слизистых начинается почти моментально, но даже близко не фактор смерти. В самом организме ничего не закипает, он, представь себе, относительно герметичный, ничего не будет высосано, даже если у тебя порез будет.

>Что если оставить человеку шлем с подачей дыхательной смеси, герметично соединенный с кожей, но оставить все остальное тельце без защиты?


Такое соединение обеспечить едва ли можно, но если сделаешь, то будет такое:
засос всего тела, распухнет, раскраснеется, стесненные движения и боль.
Моментально пропукается, но какашки останутся. Мочевой пузырь под контролем остается. Дыхание при поддержании атмосферы в шлеме будет нормальным.
Далее, если я не путаю цифры, человек умрет от теплового шока, т.к. организм даже в тени от солнца не будет успевать охлаждаться. Или я путаю, и в тени будет успевать? Я давно считал, забыл уже.
301 483744
>>3702
Спасибо.

>Ты не можешь задержать дыхание в вакууме.


Это потому, что в атмосфере давление в легких равно давлению снаружи (ну или больше лишь чуть-чуть), а в вакууме воздух из легких прет что дурной?
302 483745
>>3744
Угу, типа того.
Разницу давлений в атмосферу гортань не держит.
303 483753
>>3745
Зато анус держит
304 483754
>>3753
Таки не держит, в вакууме сразу пропукаешься.
305 483775
>>3745
А если рот заклеить скотчем и нос зажать бельевой прищепкой?
306 483794
Насколько круче продвинется астрономия, если обмазать океанские суда небольшими телескопами со старлинком для единой коммуникации?
307 483796
>>3794
Ни на сколько. Море - самое хуевое место, куда можно придумать сунуть телескоп.
308 483808
Предположим, стало известно, что Земле через 30 лет пиздец, полный и бесповоротный. Например, засекли здорового пришельца из облака Оорта диаметром в 500 км (такой хуй отклонишь), который стопудово летит курсом на столкновение с Землей. Планету на куски он вряд ли разъебет, но всю жизнь крупнее микробов наверняка похерит.

Сможет ли человечество хоть что-то сделать для своего спасения за это время или это ВСЕ? Колонии на Марсе, Луне, космические станции? Причем все должно быть автономно (хотя бы в совокупности), Земли-то не будет.
309 483809
>>3808
не
310 483812
>>3808
Нихуя не сможет даже если усрется
311 483817
>>3808
Сможет. Ведущие страны урежут военные бюджеты и социалочку и построят колонии на Луне.
312 483878
>>3817
А ты все равно сгоришь в хрущебе, так что похуй.
313 483886
>>3808

>Сможет ли человечество хоть что-то сделать для своего спасения за это время или это ВСЕ?


Изобрести телепортатор
314 483910
>>3775
Попробуй так сделать и сильно выдохнуть.. Через уши и глаза выйдет
315 483911
>>3808
Где-то читал что если сейчас упадет метеорит как тот что динозавров убил то нам нихуя особо не будет
316 483918
>>3911

>нам нихуя особо не будет


В первые пару лет да, потом из-за пошедших по пизде пищевых цепочек и нас накроет.
317 483921
>>3817

>колонии на Луне


Полностью автономные колонии невозможны даже теоретически. Чтобы такую колонию построить нужны finest technology, а чтобы колония автономно существовала, для самовоспроизводства нужен примерно миллиард человек, чтобы покрыть все необходимые цепочки от говночиста до производства микропроцессоров.
318 483935
С помощью каких естественных явлений можно придать планете большей или меньше массы, чем у Земли, вес тела на поверхности, чтобы ощущалось как на Земле?
319 483936
>>3911
Метеорит, убивший динозавров, был около 10 км в диаметре. А тут 500 км, в 125 000 раз больше энергии при той же скорости.
320 483949
Объясните, как можно наебать предел Роша, чтобы катаясь по нему, луна поовиной своей была под ним, другой - над ним, но был целёхонька?
Это должна быть только конструкция или какие-нибудь другое, более естественные варианты возможны?
321 483951
>>3935
Уменьшить радиус и увеличить среднюю плотность, кроме этого никаких.

Забавный факт: это можно и на Земле провернуть. Плотность нашей планеты настолько неравномерна, что можно срыть всю кору и мантию, и ускорение свободного падения на оставшемся ядре будет даже выше, чем на поверхности.

И наоборот, можно сверху накидать еще более тысячи километров вещества с плотностью коры, прежде чем гравитация на поверхности начнет увеличиваться.
322 483953
>>3949
Единственный способ - не быть жидким телом. То есть быть достаточно маленьким, чтобы не свернуться в кругляш как Луна, тогда предел Роша для жидких тел к тебе неприменим. И быть достаточно прочным, чтобы не развалиться внутри предела Роша для твердых тел.

А именно наебать предел Роша не получится, потому что гравитация, хоть и является самой слабой фундаментальной силой, может пересилить абсолютно все остальные, если накопить достаточно много массы. В планетарных масштабах она пересилит любые молекулярные связи, поэтому делай какую хочешь конструкцию, хоть из анобтаниума - начиная с околопланетарных размеров она свернется в клубок от самогравитации и будет подвержена пределу Роша (и будет иметь свой собственный). В звездных масштабах она пересилит вообще все остальные силы, если будет достаточно массы, и получится черная дура.
323 483955
>>3953
А если тело из вещества нейтронной звезды?
324 483969
>>3955
Вообще-то вещество НЗ такое как раз из-за самогравитации и есть.

В общем, если тело уже свернулось в кругляш, то гравитация там уже по факту доминирующая сила и все будет как она велит, в т.ч. тело будет пидорасить под пределом Роша другого тела.
1515747976382.jpg18 Кб, 451x451
325 483970
>>3955
То это, внезапно, нейтронная звезда.
326 483971
>>3970
Действительно...
327 483972
>>3971
Кто бы мог подумать.
image.png636 Кб, 980x583
328 483974
>>0719 (OP)
Как через 5-6 лет собираются вывести новую МКС на орбиту ИСЛ? У Луны же масконы, из-за чего придется часто подавать корректирущий импульс, чтоб не ебнуться на Луну или не улететь обратно к Земле.
И как будут доставлять ресурсы типа воды/еды, это же неебических денег будет стоить, ракету-носитель мощнее придется строить по идее? я мало в этом разбираюсь, прошу прощения.
329 483975
>>3974
Чтобы ответить на этот вопрос тебе нужно всего лишь посмотреть на свою же картинку, на правый край этой картинки. Там ты увидишь орбиту, которая элиптическая, а значит вблизи Луны они пролетают очень быстро и поэтому им как бы плевать на масконы. Станция у Луны это не типа МКС, это что-то подороже и не будет заселяться на постоянной основе. Как бы предполагается, что они прилетели и принесли свою жратву с собой, провели исследования на нужный срок, улетели.
330 483976
>>3974>>3975

>на орбиту ИСЛ


Не, намного хитрее - гало-орбита у L2. Для коррекции модуль с плазменными двигателями - Power and Propulsion Element - на картинке крайний слева.
https://www.youtube.com/watch?v=X5O77OV9_ek

>доставлять ресурсы типа воды/еды


Станция посещаемая, на два-три месяца, экипаж четыре человека, много и часто припасов доставлять не нужно. Существующих, а тем более разрабатываемых американских РН вполне хватит, но транспортные корабли потребуются новые.
331 483977
>>3975
Глупый вопрос конечно, а чем это будет отличаться от нынешней МКС? Ну вот сейчас космонавты в условиях микрогравитации проводят эксперименты, вкд и т.п. А нахуя собственно то же самое проводить вблизи с Луной? Только ради изучения поверхности?
332 483979
>>3977
МКС по сути даже не в настоящем космосе, микрогравитация это как свободное падение, она защищена от всего магнитным полем Земли, до неё долетают водоросли из океана, плазма и газы атмосферы Земли, купается в радио.
Если проводить любой эксперимент у Луны, может дать другой результат.
333 483986
>>3974

>У Луны же масконы, из-за чего придется часто подавать корректирущий импульс, чтоб не ебнуться на Луну или не улететь обратно к Земле.


Во-первых, это относится только к достаточно низким орбитам. Во-вторых, у Луны есть несколько замороженных орбит, которые очень стабильны. https://en.wikipedia.org/wiki/Frozen_orbit#Lunar_frozen_orbits . Ну и да, не на низкую олололунную собираются выводить, как анон написал. С Луной вообще дохуя вариантов орбит, некоторые как LDRO довольно нетривиальные.

>>3977
В предыдущей ипостаси олололунная станция задумывалась еще и как перевалочный пункт для стыковки поезда на Марс, и для неё предлагалась удаленная ретроградная орбита (LDRO), с которой очень удобно стартовать межпланетный перелет. В текущем варианте с NRHO не знаю какие профиты.

>>3979
Экспериментов, точно требующих прямо межпланетных условий, я что-то и не припомню. А вот то что космонавты на станциях топают, вентиляторы гудят и нарушают вибрациями невесомость - это гораздо более серьезная трабла, отсюда и рисуют автоматические коорбитирующие модули вроде Ока-Т2, не связанные жестко со станцией.
334 483987
>>3979

>до неё долетают водоросли из океана


Когда кстати намечается завершение этого эксперимента? Вроде ж результат экстраординарный, это ведь потенциальная панспермия, епта. Но что-то я не вижу фурора и кучи работ по ионному лифту или любому другому механизму поднятия этого ебаного планктона на сотни км. Может все не так радужно как в промежуточных результатах?
335 483988
>>3986

>Экспериментов, точно требующих прямо межпланетных условий, я что-то и не припомню.



Извини, анон, но ты не можешь помнить и знать всё, и это не значит что таких экспериментов нет, или что их нельзя придумать, чтобы потом написать научную работу, или разработать грант, или испытать технологию/материал с припиской "приспособлен для работы в глубоком космосе".
336 483989
>>3986

>В текущем варианте с NRHO не знаю какие профиты


Такие же - достаточно пукнуть перилуне, чтобы прыгнуть в л1 или л2, выйти на элиптическую геоцентрическую, ну и к марсу там. Правда ассемблить корабль в ебенях у луны, чтобы оттуда стартовать к марсу задачка той еще тупизны, ни разу на таких орбитах не стыковались вообще в истории.
338 483991
>>3990
Да я в курсе, еще вот
http://knts.tsniimash.ru/ru/site/Experiment_q.aspx?idE=191
по результатам которого и отчитывались об обнаружении планктона на внешних поверхностях МКС и некоем механизме переноса микроорганизмов с земли в космос. Заканчивается якобы в 2020. Мне просто странно что никакой особенной движухи на эту тему нет, отрепортили разок и все, это ж пиздец находка по идее.
339 484005
>>3955
>>3969
А допустимо ли в теории существование нетронных звезд меньшей массы чем обычно? Например, в результате распидораса другой нейтронкой.
340 484007
>>3979

>МКС по сути даже не в настоящем космосе,


Пожалуйста, больше не отвечай в этом треде. По крайней мере так.
Единственный верный отличительный фактор ты упомянул - это магнитосфера.
А "микрогравитация" - она для всех намерений и целей - невесомость как есть.
341 484008
>>3991
>>3990
>>3987
Вы будете смеяться насколько дохуя МКС предвносит в науки и насколько всем ПОХУЙ.
Все хотят смотреть как 22 мужика пинают шар 90 минут или те же 90 минут смотреть на то как фиолетовый мужик распидорашивает волшебных героев.
Взять бабки от этого БЕСПОЛЕЗНОГО говна и перевести в космос - мир бы другим был.
342 484021
>>4007
На самом деле он частично прав, у длительных экспериментов на экспозах-р например были те же проблемы с внешней атмосферой и с кислородом в частности. Но основной фактор, гробящий потенциальные эксперименты, это плохое качество невесомости, особенно кристаллографию и большинство биологии.
343 484024
>>4021
Само собой.
Но это аспекты не связанные с невесомостью
Для экспериментов в невеомости МКС прекрасно справляется.
344 484031
>>4024
Если ты о градиенте гравитации, из-за того что МКС находится на слишком низкой орбите - это действительно минимальный фактор, но только потому что есть вибрации, которые на порядки больше. Если избавиться от вибраций (запилив маленькую самостоятельно летающую бочку для экспериментов, по возможности вообще без движущихся частей), то окажется что и градиент на этой орбите слишком велик для некоторых экспериментов в молбиоле и кристаллографии.
345 484033
>>4031
Безусловно, есть эксперименты для которых надо минимальное гравитационное воздействие.
Тем не менее - большая часть экспериментов могла бы быть выполнена вообще на вомит-комете если бы тот мог в разы дольше сигать. Часы, дни.
346 484045
>>3935
Раскрутить до скорости пульсара.
347 484049
>>4045
Че несешь
Без огромной гравитации это даст выкидывания всего в космос, в том числе кусков планеты
348 484073
>>4049
Значит раскрутить до меньшей скорости, чтоб гравитация была околонулевой, но планету не разрывало.
349 484077
>>4073
но тогда гравитация будет неравномерна, на экваторе низкая, на полюсах как обычно
350 484081
>>4077
Тоже неплохо.
352 484086
>>4085
Че сказать хотел, "умник"?
Подавляющее большинство зеро-г экспериментов отлично проводится в МКС.
353 484087
>>4086
Не-а, всегда к ним можно добавить *(проведено на МКС в условиях микро-гравитации). И любой человек потом может поставить точно такой же эксперимент в 2млн км от Земли и сказать, что вот он-то провёл его в условиях настоящего глубокого космоса и невесомости.
354 484100
>>4073
Не выйдет, по той же причине что и у анона с пределом Роша выше. Планета держит форму за счет самогравитации, а ты ее раскручиваешь, работая против гравитации. Планету будет плющить по экватору, пока не разорвет, и предел скорости вращения довольно небольшой.
355 484116
>>3936
Ну хуй знает, метеорит как миниспутник.. сомневаюсь что такие есть
356 484122
>>4116
Так а никто не знает, что там в туманности Аорты облаке Оорта есть. Тут транснептуновые объекты с трудом открывают, а облако Оорта тянется до 50-100 тысяч а. е.
357 484123
>>4087
И он получил бы такие же результаты. Проблемс?
358 484188
Разве САС может успеть увести корабль от взрывной волны взрывающейся ракеты? Ударная волна должна успеть ебануть по кораблю и сломать его
image.png708 Кб, 800x663
359 484191
>>4188
САС нужна чтобы увести корабль до взрыва, при появлении неполадок.
А вообще при аварии Челленджера кабина была относительно целая после взрыва, все умерли от удара об воду.
360 484193
>>4191
Их бы спас парашют?
361 484196
>>4193
Скорее всего. Есть даже байки что они были полностью в сознании и пытались прыгнуть с парашютами из люка
362 484197
>>4196
Было б охуенно увидеть видео из кабины...
363 484211
>>4188
Ракета при распидорасе просто сгорает, а не детонирует как бомба. Бабах получается большой, но медленный, как в голливудском кино. К тому же работает против набигающего воздуха. Так что можно успеть упиздовать с огромным ускорением.

>>4191
САС и от взрыва должна уносить, на самом деле.
364 484213
>>4211
Когда протон ебнул - взрывная волна довольно громко бабахнула как от взрывчатки
365 484215
>>4123
Та не...какие-нибудь квантовые кристаллы показали бы отклонение, атомные часы показали бы отклонение на триллион лет.
366 484217
>>4213
Ясен хуй, там же дохуя тонн горючки. Но громкость это одно, а скорость распространения взрыва это другое.
367 484218
>>4215
Для всех намерений и целей это несущественно.
368 484219
>>4217
Если за километры взрывная волна стекла выбила, то что бы она с близкой капсулой сделала бы?
369 484220
>>4217
Но скорость распространение при резком смешивании гиперголиков не должна быть выше смешивания компонентов с внешним зажиганием?
370 484221
>>4218
Как сказать, как сказаит
371 484225
Эксперименты которым невесомость нужна получаются на порядки лучше чем на земле, это факт.
372 484250
>>4219
Тряхонуло бы
373 484266
Объясните мне, как частицы в радиационном поясе двигаются - по орбите круговой как спутник, или вдоль силовых линий как камикадзе быстро сдыхая в атмосфере, потому что силовая линия замыкается вблизи полюса? Почему говорят тогда что в этих поясах частицы живут много лет? Какая-то постоянно конфликтная информация идёт, какие-то картинки цветастые гуглятся и ничего не понятно.
image0154.jpg25 Кб, 514x260
374 484275
Особенно мне вот эта ебота интересна - ТОЧКИ ОТРАЖЕНИЯ. Следите за движениями стрелок. Якобы частица может по магнитной линии спиралью лететь вниз и вот уже скоро она столкнётся с атмосферой, но потом она такая - ЛОЛ, Я ПОШЛА ОБРАТНА - и совершает обратное движение до следующей границы атмосферы, а потом опять магнитная сила даёт ей магический и билет и она - ЛОЛ, Я ПОШЛА ОБРАТНА. Что это за хуета блядь? Кто там в точках отражения раздает частицам билеты на бесконечные путешествия от полюса к полюсу? Что за Гэндальф там сидит и поворачивает частица назад? Нахуя мне мозги KSP испортили, чтобы потом вот такими трюками бесить?
375 484293
>>0719 (OP)
Посоветуйте годную книгу про космическую технику, про опыт полетов космонавтов/астронавтов на МКС, мир и т.п.
Криса Хэдфилда уже читал, очень душевный человек
376 484295
>>4293
Сам не читал, но вроде хвалят "Верхом на ракете" Майка Маллейна (Riding Rockets by Mike Mullane) за эдакое "покровосрывательство".

https://royallib.com/book/Mullane_Mike/Riding_Rockets.html (eng.)
https://www.e-reading.club/book.php?book=1051663 (рус.)
377 484297
>>4295
Большое спасибо
и отдельное спасибо за ссылку
378 484304
>>4293
Космос. Прошлое, настоящее, будущее - Сурдин Первушин Левитан Мамуна
379 484318
Анон какая гипотеза тебе кажется более вероятной

1. Звезды темной энергии
2. Гравастар
3. Черная дыра
380 484323
>>0719 (OP)
4то насчет запуска в космос объектов при помощи рельсовой пушки? без экипажа, без топлива снаряд на орбиту, где он сбрасывает оболочку и выполняет свои космические задачи.
381 484324
>>4323
Атмосфера мешает.
Внутри атмосферы на орбитальной скорости нельзя летать - СГОРИШЬ к хуям.
382 484335
>>4318
Планковская звезда.
383 484336
>>4323
На Луне это возможно. На Земле или Марсе - нет.
384 484345
>>4324
А если в холодильнике?
385 484346
>>4345
Шо
386 484347
>>4008
Меня тоже постоянно бомбит насколько человечество нерационально расходует ресурсы
387 484348
>>4345
Энергии невъебенно потребуется
388 484352
>>4345
Тогда ещё и холодильник сгорит к хуям.
389 484384
Наша Вселенная — эмуляция?
390 484385
>>4384
Никто не знает и знать не может. Все попытки доказательства за или против построены на (безосновательных) предположениях о природе среды, в которой выполняется эмуляция.
391 484399
>>4384
Скорее всего, да. Не обязательно в том смысле, что нас эмулируют "архитекторы матрицы".
392 484404
>>4399

>Скорее всего, да.


На основании чего такие выводы?
393 484406
При слиянии чёрных дур гравиволны могут быть столь сильными, что смогут растаскивать виртуальные частицы друг от друга и делать их реальными? Чтоб это слияние ебически светилось тогда, хотя бы вблизи.
394 484407
>>4404
На основании последних результатов теоретической физики относительно вселенных с положительной космологической "постоянной".
395 484408
>>4406
Черные дуры и без всякого слияния светятся
396 484409
>>4406
Теоретически да, но для черных дыр звездных масс, чьи слияния наблюдают IRL, этот эффект пренебрежимо мал, а характерные длины волн получившихся фотонов буду порядка сотни км.
15582606946530.mp44,1 Мб, mp4,
384x480, 1:00
397 484460
Я дегенерат и не понимаю - почему не взлетит?
398 484461
>>4460
Что не взлетит? Ты тут очередной вечный двигатель увидел вместо развлекательного видева на вечер под пивас?
399 484462
>>4460
Ознакомься
https://youtu.be/15V0gUXUPko
Там не только про это решение.

Вкратце, если в язык не можешь: это кручение работает только когда ты держишь рукой магниты, если стабильно установить - ничего не будет. Из-за неравномерного держания они движутся. При установке на место найдется стабильная позиция и ничего не будет двигаться.
400 484463
>>4462
Т.е. всё из-за микродвижений руки?
А если юзать энергию на дестабилизацию, например магниты на пружину прикрепить и вращение бы закреплённые магниты бы двигало немного.
401 484464
Test
402 484465
>>4463
Энергия из ниоткуда не берется.
Даже когда ты рукой магнит тилибонькаешь у тебя выдается мизер достаточный лишь чтобы светодиод запитать.
403 484495
У меня вопрос. Почему правительство США подрядило Spcace X создавать ракеты если сами наса могут сделать то же самое, но дешевле?
404 484498
>>4495

>У меня вопрос. Почему правительство США подрядило Spcace X создавать ракеты если сами наса могут сделать то же самое, но дешевле?


Потому что сенатская комиссия в свое время выяснила что спейсх создает ракеты в 2.5 дешевле НАСА. В том числе потому что НАСА когда создает ракеты все равно покупает компоненты у коммерческих подрядчиков типа боина и дорого.
405 484500
>>4495
NASA не производит ракеты самостоятельно. Производством в основном занимаются подрядчики. Например Rocketdyne, North American, Rockwell, McDonnel, Douglas и т.п. (сейчас большинство слилось в несколько крупных контор вроде Боинга). В NASA только разрабатывают ракеты, лишь некоторые и то не целиком зачастую.

>правительство США подрядило Spcace X создавать ракеты


У тебя каша в голове. SpaceX - одновременно разработчик РН, их производитель, и оператор их запусков, сдающий свои запуски "под ключ", от заказчика требуется лишь нагрузка. Falcon, основная их рабочая лошадка, создавался по собственной инициативе, было несколько предполагаемых конфигураций, из которых выжил только Falcon 9, потом появился Falcon Heavy. Уже после создания F9, когда SpaceX продемонстрировала способность создать полноценный носитель, NASA передали SpaceX некоторые технологии (в основном результаты испытаний), что упростило дальнейшую разработку. (передача технологий, профинансированных из федерального бюджета, в частные руки является их первоочередной задачей, согласно соответствующему акту 1958 года).

NASA подрядило SpaceX запускать нагрузки к МКС например, если ты об этом. Причём SpaceX - не единственный подрядчик NASA и скорее всего никогда единственным не будет, т.к. класть яйца в одну корзину считается ненадежным. Есть Боинг, есть Орбитал и т.п.

>могут сделать то же самое, но дешевле


Не могут, это одна большая бюрократическая система с минимальным стимулом, типичные подрядчики тоже не вылезают из госконтрактов и не имеют стимула развиваться. Акт о частной космонавтике 1985 года был принят как раз из-за осознания проблемы в конце 70х, и из-за стремления повысить конкуренцию в отрасли. Акт 2004 года, продвинутый как раз во многом SpaceX - его улучшенная версия, позволившая сдвинуть частную космонавтику с мертвой точки.
406 484501
>>4500

>Falcon, основная их рабочая лошадка, создавался по собственной инициативе


По COTS же. Объявили о РН раньше, но фактически разработка началась после заключения контракта с NASA.
407 484512
>>4501
F1 разрабатывали на свои. COTS был уже после того как конфигурация F9 устоялась как единственная, а был еще F5, на который не нашли спроса, но если бы нашли то делали бы, и разные другие.
408 484519
>>4500

>Не могут, это одна большая бюрократическая система с минимальным стимулом, типичные подрядчики тоже не вылезают из госконтрактов и не имеют стимула развиваться.


Тут следует добавить, что штатный вариант оплаты таких контрактов проводится по схеме "затраты + %прибыли", что сильно не способствует снижению расходов, поскольку за превышения не только не штрафуют, но еще и прибыль растет, а единственное что ограничивает аппетиты - суровые дяди из конгресса, которые гипотетически могут черезмерно разбухшую программу порезать.
А вот с ПейсИксом и прочими "новыми частниками" идет переход на человеческую схему с фиксированной стоимостью контракта.
409 484535
Парни, какую программу спасения жителей города в корабле поколений можно придумать, на случай отказа икусственной гравитации: ведь все океаны начнут вылетать, вода растечётся, все погибнут?

По моему выходит один: запрет всех водоёмов на поверхности города, без шлюзов - блокировок.

То есть на пикрелетейде не будет никогда воды и столько незакреплённых- объектов.
Понятно, что можно сделать вращение надёжным и создать сотни запасных термоядерных реакторов, но мы же знаем что всегда что то может пойти не так и техногенная катастрофа уровня отключения гравитации убъёт всех жителей корабля.
410 484537
>>4535
Маняфантазии в /sf/
411 484538
>>4537
Сделай мне /hsf
В sf сам иди.
412 484539
>>4535
Твою космическую станцию можно будет обляпать тоннами 3D графики и прочими голограммами или как там это называется, которые будут создавать вокруг иллюзию хоть карибского моря с пальмами и песочными пляжами, еще и звуковыми эффектами сопровождать и запахами соответствующими, и при этом в случае чего не полетят в свободное плавание. А реальное устройство и интерьер делать достаточно практичным и функциональным без дополнительной мишуры.
413 484544
>>4535

>отказа икусственной гравитации


>вращение


Если ты такую бандуру раскрутил, "отказа" не будет. Просто так момент вращения никуда не денется, и поддерживать его не надо. Он сохраняется ровно так же, как и поступательное движение. Или Земля, по-твоему, тоже постоянно движком вращается?

Чисто теоретические сценарии отказа:
1) Вращение очень медленно останавливается, видимо из-за трения (ага, корабль поколений полетел в атмосферу прохлаждаться). В любом случае, времени на откачку воды (хоть в резервуары, хоть за борт) будет навалом.
2) Вращение останавливается резко. В таком случае, летающая вода и захлебывающиеся обитатели - последнее что тебя должно волновать. Ибо такая остановка возможна только при гигантской приложенной энергии, которая один хуй сломает к чертям все несущие конструкции.
414 484549
>>4544
Кстати, наличие у Земли ядра в том виде в котором оно сейчас есть сказывается на скорость вращения?
415 484550
>>4549
Ну в некотором смысле да, сказывается. Если при сохранении момента инерции (момента вращения) магически перераспределить плотность Земли до равномерной, вращение замедлится.
416 484553
>>4535
Реальный корабль поколений будет такого размера и массы, что не будет нужды создавать искусственную гравитацию, будет вполне себе естественная.
417 484554
>>4553
бля, ну не начинай
418 484555
>>4553
Нет. У Фобоса диаметр порядка 20 км, а гравитация на поверхности - менее тысячной g. Плюс в корабле поколений, по идее, живут внутри, где его естественная гравитация еще ниже. Да и плотность у него будет низкая, пушо никто не будет делать пятиметровые переборки из цельного чугуния: эту байду еще как-то разогнать же надо.
419 484556
>>4555
Фобоса для корабля поколений явно не хватит.
420 484557
>>4556

>явно


Мерять нынешними технологиями вообще бессмысленно, так как сейчас такое банально невозможно вообще. А что там в теории изобретут в далеком будущем - хз. Так-то 20 км - это диаметр немаленького города. Плюс город-то более-менее плоский, а Фобос - трехмерный, то есть по объему он как сотня городов. Понятно, что нужны еще площади под всякий жрач, но кто знает, может к тому времени будет доступен холодный термоядерный синтез бифштексов прямо в микроволновке.
421 484558
>>4557

>микроволновке


А микроволновка на корабле поколений по воле Б-жьей материализуется? А если микроволновка сломается, или двачер будущего выкинет её в окошко? Выходит на корабле придётся найти место для завода микроволновок, а заодно и людей которые будут на нем работать. И людей, которые будут учить людей, которые будут работать на заводе микроволновок.
sage 422 484560
>>0719 (OP)
Допустим, высокоразвитая цивилизация построит сферу дайсона у какого-нибудь Сириуса и будет посредством лазера или зеркал передавать 99% энергии в сторону Проксимы Центавра. Будет ли виден этот луч света с Земли?
423 484561
>>4558
Ну опять же, это с позиций нынешних технологий. Может у них будут самоорганизующиеся нанороботы из сплава анобтаниума с нанотрубками с гарантийным сроком службы 10000 лет. Фантастично, невероятно, но теоретически законам физики не противоречит. Хуй знает, что там будет через пару тысяч лет. Твой смартфон в Древнем Риме тоже магией бы выглядел.
424 484562
>>4561
Вся история человечества показывает, что с усложнением экономики (производственных цепочек) необходимое количество человек вовлеченных в эту экономику растёт. В неолите автономно могло жить племя в несколько десятков человек, в античности город-государство на несколько тысяч человек, в средневековье княжество на несколько десятков тысяч человек, и так далее. В технологических условиях 20 века для автономного существования нужны уже сотни миллионов. Нет никаких причин предполагать, что в будущем эта тенденция прекратится.
425 484564
>>4562
Во-первых, возможна и даже вероятна дальнейшая автоматизация труда с развитием ИИ. Вплоть до почти полной. Поэтому миллиарды человек не понадобятся, большинство их мест займут куда более компактные, эффективные и самообслуживающиеся роботы (мы ж говорим о далеком будущем, ну).

Во-вторых, у такого корабля нет задачи стать вечным домом. Он должен фунциклировать очень долго, но все же конечное время (скажем, 10000 лет), до достижения планеты-цели. С теперешними технологиями 10000 лет это практически вечность, значит все нужно мочь и уметь производить на месте. В будущем, теоретически, могут быть разработаны такие технологии, что 10000 лет станут просто нормальным сроком эксплуатации. И производить уже не нужно будет, достаточно обслуживать и ремонтировать (с помощью тех самых роботов) имеющееся.
426 484567
>>4564

>Во-первых, возможна и даже вероятна дальнейшая автоматизация труда с развитием ИИ


Вся история человечества это автоматизация труда. Но каждый шаг автоматизации позволяет занять высвободившихся людей чем-то новым,на этом этапе совершенно необходимым, чего предыдущим поколениям и на ум бы не пришло.

>10000 лет станут просто нормальным сроком эксплуатации


Значит на ресурсные испытания тоже уйдет 20-30к лет. Это уже какая-то совсем ненаучная фантастика.
427 484569
>>4567

> Значит на ресурсные испытания тоже уйдет 20-30к лет.


Ну да блядь, а когда на автомобиль дают три года гарантии это значит что перед этим его три года испытывают, да?
Хуйни не неси.
428 484570
>>4560
Нет.
429 484574
>>4544
Я предполагал умеренно резко сокращение вращения, например, от удара метеорита и заклина какие то креплений, поломки крепления, создание ассиметрии вращения, выхода реактора из строя и так далее. Она ведь за день может остановится вполне.

Земля в этом плане надёжней, её реально не остановить, потому что она вращается без всякого трения, да, и имеет дохуя большую энергию.
430 484598
Где искать телескопотред?
431 484599
>>4598
В Томске.
432 484600
>>4598
Это наверное самый тупой вопрос что я видел в этом треде, "почему всплывают пельмени" на втором месте
433 484601
>>4599
>>4600
Семён, успокойся.
434 484602
>>4600
А почему пельмени то всплывают?
435 484603
>>4601

> пукпукпук симен


Лол. Накатим?
436 484604
>>4602
Потому что вода внутри превращается в пар а пар легче воды.
437 484605
>>4604
ебать дибил
438 484606
>>4605
Нет ты.
439 484609
>>4569

>Ну да блядь, а когда на автомобиль дают три года гарантии это значит что перед этим его три года испытывают, да?


>


Примерно так, только не весь автомобиль целиком, а системы входящие в его состав по отдельности. Да что там говорить, даже в икее проверяют ресурс петель выдвижных ящиков именно открывая и закрывая его десятки тысяч раз.
440 484618
>>4574

>Я предполагал умеренно резко сокращение вращения, например, от удара метеорита


Удара метеорита достаточной силы, чтобы остановить вращение, разнесет станцию в клочки.

>Она ведь за день может остановится вполне.


За какой день? Чтобы она останавливалась день, нужно чтобы весь день на нее действовала какая-то сила, и это не удар метеорита (который моментальный). Какую силу ты можешь предложить для этого?
441 484619
>>4618
Метеоритная защита не позволит напрямую ударить в город, но она может быть повреждена, реакторы могут выйти из строя, и за счёт трения (та самая постоянная сила) это ебень спокойно будет тормозиться.
Чтобы выкачать воду нужно 7 дней.

Твои действия?

Выкачанная вода будешь напораживаться на трубках и создавать заторы, роботы будут неуспевать отбивать сосульки.
Слушай, сцеанрией дохуя почему это йоба может начать останавливаться, начиная от потери тяги, заканчивая диверсией в тех. блоке.
442 484620
>>4619
Какого трения, шизик, с чего бы центрифуге о что-то тереться?
443 484621
>>4620
Трения об эфир, наверное.
444 484624
>>4620
в любой системе будут возникать небольшие ассиметричные круговые моменты, из за неравномерности распределения массы в крутящемся городе, наклон оси будет создавать трение о крепления, которые придают ему скорение и стабилизируют ассиметрию. В случае отключения захвата, за небольшое время накапливается небольшая ассиметрия вращения, что выходит за допустимые в эксплатации рамки, оставить же крутяжиеся города вращаться в зажиме крепления создаёт трения.
Реакторы вышли из строя, энергии не хватает, гравитация пропадает, вода из морей начинает представлять смертельную опасность. Слив воды слишком медленный, так как решение принималось советом два дня. Ну ты понял.
445 484625
>>4624
Понял. У тебя какой-то свой чудный манямирок.
В сайфаче спроси за него.
В реальном мире придуманные круговые хуйни ни с чем не трутся
446 484626
>>4625

>Понял. У тебя какой-то свой чудный манямирок.


Спасибо, добро пожаловать как говориться.
Всегда готов принять пару гостей в моём чудном манямирке.
447 484634
>>4609
Охуеть, неужели кто-то работает выдвигальщиком ящиков?
448 484635
>>4634
Роботы.
449 484636
>>4635
Отнимают работу у человеков блядские роботы.
450 484637
>>4636
Если у тебя квалификации хватает только на выдвижение ящиков, то может быть ты и заслуживаешь чтобы роботы забрали твою работу.
451 484638
>>4637
А ты попробуй блядь сто тыщ раз открыть-закрыть ящик, посмотрю хватит ли у тебя квалификации, червь.
Стикер512x512
452 484639
>>4638
Я для этого соберу робота.
453 484640
>>4636
https://www.youtube.com/watch?v=UOzrh9zk9hc
А ну иди сюда, кожаный мешок
pepe thinking.jpg29 Кб, 640x519
454 484641
>>4640
Эти штуки надо для мастурбации приспособить.
455 484642
>>4641
Бабу лучше приспособь.
456 484644
>>4642
С бабой любой дурак может, а вот с роботом - мало кто.
457 484648
>>4640
Бля, это долбоебы вместо того чтобы сделать простейший кривошипно-шатунный механизм захуячили ремень блядь с реверсом.
458 484649
>>4648
Это же шведы.
Стали бы они делать механизм в котором КРИВОШИП и ШАТУН?
459 484650
>>4648
Могут себе позволить.
460 484651
>>4560

>Допустим, высокоразвитая цивилизация построит сферу дайсона у какого-нибудь Сириуса и будет посредством лазера или зеркал передавать 99% энергии в сторону Проксимы Центавра. Будет ли виден этот луч света с Земли?


>


Если на его пути попадется облако пыли\газа, то увидит.
461 484652
Что будет с гравитацией под поверхностью, на поверхности и на некотором удалении от поверхности объекта, который представляет из себя пустотелую сферу, диаметром и массой примерно равными Земле, но с толщиной стенок в 1 см? ну или чуть большая толщина, такая чтобы не образовывалась ЧД, нейтронная звезда и прочая экзотика.
462 484655
>>4652

>пустотелую сферу, диаметром и массой примерно равными Земле, но с толщиной стенок в 1 см


импоссибру
выбери два параметра
image.png268 Кб, 480x360
463 484657
>>4655

>импоссибру


А если так?
464 484660
>>4652

>под поверхностью


Внутри - полная невесомость, в "толще" стенок линейно убывает с глубиной от 1 g до 0.

>на поверхности и на некотором удалении от поверхности


Такая же как на Земле.
465 484663
>>4500
>>4498
Спасибо за ответы. У меня ещё вопрос. Почему компания тесла убыточна, ведь они производят хорошие авто, пользующиеся спросом во всём мире?
466 484667
>>4663
Потому что продают ниже себестоимости.
467 484739
почему орбита вокруг земли до сих пор не забита под завязку зеркалами?

зеркало из отражающей плёнки это очень простое в инженерном плане устройство, тем более в условиях свободного падения.

профитов не счесть - можно ночью освещать поля растений чтоб они быстрее росли, можно делать города потеплее зимой, а так же продлевать световой день, можно даже при наличии очень большого количества зеркал разрушать или не давать формироваться ураганам и торнадо, нагревая определённые участки атмосферы в определённый момент, можно круглые сутки светить на поля солнечных батарей для выработки электроэнергии кругые сутки, можно дополнительно подсвечивать и днём и ночью поля солнечных батарей в высоких широтах чтоб больше электричества вырабатывали, можно растапливать проходы в ледяных полях в северном ледовитом океане для прохождения грузовых судов без ледоколов

короче можно миллионы применений придумать
468 484752
Если гравитационные волны это разновидность эм-взаимодействия и они распространяются со скоростью света, то чому их не засасывает в чд как обычный свет?
469 484754
>>4752
Они не разновидность ЭМ-взаимодействия.
470 484755
>>4739
Сдует/упадут/нахуй не надо
471 484756
>>4739
Хуясе очень простое, а как ты будешь его раскрывать, поддерживать плоским и поворачивать в нужную сторону? Выкинуть мешок пленки на орбиту и правда несложно, но она просто соберется обратно в комок, и уж тем более не будет светить куда нужно.
472 484758
>>4754
Чому тогда распостраняются со скоростью света?
473 484759
>>4758
Это максимальная скорость взаимодействий в принципе.
Свет лишь одно из них которое ограничено этой константой и первое для которого такое ограничение обнаружено.
474 484760
>>4739
Пассивное зеркало - космический мусор. Чтобы отражать куда-то, надо ориентировать на солнце и точку, нужно удерживать ориентацию от дрифта из-за гравитационных возмущений и сопротивления атмосферы, нужно пополнять группировку либо бустать зеркало повыше, как опустится. В общем нужна активная система ориентации, как минимум. По сути полноценный космический аппарат получается.

>тем более в условиях свободного падения


В них наоборот намного сложней. Все что можно вывести на орбиту дешево - это тонкие раскладные конструкции. Они постоянно колышутся, от гравитационных возмущений Луны и Юпитера (внезапно), от неравномерности гравитации Земли, от ударов микрометеоритов, от остатков верхней атмосферы, хуйни, малафьи. А еще у них есть собственная частота колебаний, и они могут войти в резонанс. Вот эта вся хуйня сильно ограничивает размеры конструкций, например тех же СБ или антенн, поэтому например до Юпитера и Сатурна АМС летают на ритэгах - СБ выйдут слишком большими (хотя казалось бы, "хули там, в невесомости"), исключение составила Juno, где их влепили от безысходности. Солнечные панели МКС это неебаться сложная конструкция. 100-метровая параболическая антенна на USA-223 это уникальное засекреченное инженерное чудо за $2млрд.

Насчет профитов тоже хуй знает с непредвиденными последствиями.
475 484770
>>4760

>Чтобы отражать куда-то, надо ориентировать на солнце и точку,


Причем еще и постоянно вертеть всю эту хрень, учитывая, что виток орбиты - всего полтора часа. А не просто корректировать раз в месяц.
476 484774
>>4760

>поэтому например до Юпитера и Сатурна АМС летают на ритэгах


А не потому что там солнечная постоянная в сотни раз меньше?
477 484775
>>4770

>иток орбиты - всего полтора часа


Низкой ололоземной. С которой спутник большой парусности сходить будет за считанные годы.
А ГСО, например, все сутки.
А можно и дальше.
478 484776
>>4775
Чем дальше твое зеркало, тем меньше от него толку, если оно не фокусирующее. А фокусировать на таком расстоянии, да с движущейся целью - пиздец какая нетривиальная задача.
479 484777
>>4776
Разместить на луне.
480 484781
>>4774
Именно поэтому, я же и говорю - нужны большие СБ, которые ограничены и массогабаритами, и колыханием. Хотя у Джуно не самые большие батареи, самое большое это наверно МКСовская ферма P6, там стоят ебические 34-метровые полоски, раза в 4 больше. Но ставить такую хрупкую хернь на АМС, которая маневрирует и летает в глухие необслуживаемые ебеня, минимум неразумно, да и тяжеловато.
481 484801
>>4752>>4754
Это нерешённый парадокс, про который я уже давно тут пишу.
В рамках текущей модели совершенно похуй на вид взаимодействия - любое взаимодействие является короткодействием, которое переносится виртуальными бозонами. Кривизна пространства их замедляет, из-под горизонта событий никакие бозоны не могут выйти в принципе. И гравитоны не могут. И всякая информация не может.

Т.е. если горизонт событий единожды где-то образовался во вселенной - он там вечность стоять должен. Первый майндфак: если, например, ЧД и звезда вращаются вокруг общего центра - значит ЧД как-то передаёт звезде информацию о своём местоположении. Второй майндфак: если взять, например, дохуя антиматерии и кинуть в ЧД - она аннигилирует, и горизонт событий сдуется, а информация об этом дойдёт до внешнего наблюдателя.

Следовательно, гравитация каким-то образом иммунна к гравитационному замедлению времени, а значит - локально сверхсветовая. Значит, сигнал из-под горизонта событий в принципе возможен. Например, если мы войдём под горизонт событий сверхмассивной ЧД с двумя маленькими нейтронными звёздочками в багажнике и будем их теребить морзянкой - гравитационные волны будут излучаться, и мы сможем телеграфировать внешнему наблюдателю о том, что же находится внутри ЧД.

Физически это можно объяснить тем, что гравитон (или любая другая передающая гравитацию хуета, какой бы она ни была) является сверхбрадионом, и его критическая скорость нихуя не C, а зависит от напряженности гравитационного поля.
482 484802
>>4801
Гравитоны не доказаны, у них матаппарата даже толком нет.
Текущее предположение что гравитация - сугубо искажение метрики пространства, никакими частицами не переносится, а потому запросто работает с ЧД.
483 484803
>>4801

>любое взаимодействие является короткодействием, которое переносится виртуальными бозонами


С точки зрения ОТО, гравитация по сути не является взаимодействием, а вообще искривлением пространства. Да, у ОТО тоже есть нестыковочки, поэтому считать ее истиной в последней инстанции не стоит, но так уж получилось, что пока ничего лучше не придумали. Поэтому, прежде чем утверждать, что гравитация - такое же взаимодействие как другие, для которых есть замечательные квантовые теории, нужно бы построить теорию квантовую гравитации хотя бы столь же точную как ОТО.
484 484805
>>4802>>4803
Нет, это так не работает.
ОТО стоит в том числе на принципе причинности. Он подразумевает, грубо говоря, что у всего есть начало, конец и причина, и что не-близкодействий в природе не бывает.

Возникает вопрос, из чего тогда сделано пространство? Если это не пустота, а хуйня с осязаемой метрикой, способная переносить информацию - значит оно сделано из каких-то подструктур, взаимодействующих друг с другом по правилам известного нам физического мира. Может быть это вообще не бозоны, а какие-то пузырьки N-мерной браны, но ведут они себя определённо подобно тому, как бозоны передают свет, а молекулы передают звук.

В этом случае у нас тут в любом случае претензия на сверхсвет. Если пузырькам похуй на эффекты пространства - значит при определённых обстоятельствах ограничения на передачу информации внутри пространства на них не распространяются. Если никаких пузырьков нет, и у нас тут чистое дальнодействие - значит его скорость в теории вообще бесконечна.
485 484809
>>4805

>какие-то пузырьки N-мерной браны, но ведут они себя определённо подобно тому, как бозоны передают свет, а молекулы передают звук.


Какого хуя всё должно опираться на чём то, и что то на чём то и где этому конец?
486 484810
Волны возникают за пределом горизонта событий, в чём парадокс?
487 484811
Максимально тупой вопрос. Каково устройство естественного термоядерного реактора внутри солнца? Тупо шар булькающей плазмы, или какая-нибудь YOBA контринтуитивной формы?
488 484812
>>4811
Куча водорода давит сверху так сильно, что атомы водорода совершают термоядерный синтез
Синтез в свою очеред давит в обратную сторону, создавая равновесие
489 484813
>>4811
Синтез идёт только в ядре. Ядро это "тупо шар булькающей плазмы" и довольно однородно по структуре, считается что ярко выраженных слоев там нет. Там идут две реакции - протон-протонный синтез и немного CNO-синтез (углерод-азот-кислород). Скорость реакции саморегулируется процессом, который описал анон выше >>4812 : повышается скорость - ядро распирает наружу - скорость уменьшается.
490 484817
>>1950
Сила тяжести пропорциональна квадрату расстояния, а расстояние от Земли до Луны соизмеримо с размером Земли. Поэтому сторона Земли повёрнутая к Луне притягивается к ней сильнее, чем противоположная. С точки зрения Земли это воспринимается как сила, растягивающая Землю в противоположные стороны (представь пружину между двумя машинами - обе едут вперёд, но та что сзади едет медленнее - с точки зрения пружины её будет растягивать в стороны)
491 484824
>>4801

>дохуя антиматерии и кинуть в ЧД - она аннигилирует, и горизонт событий сдуется


Чойто он блять сдуется? С какого хуя энергия аннигиляции должна покинуть объект?
492 484825
>>4824
И-излучение Хокинга?
493 484826
>>4825
Ясна панятна. Физика уровня 8"г", вопросов больше не имею.
15181895637550.jpg1,7 Мб, 3200x2400
494 484842
Нахуй изучать космос, впиливать столько бабла в непрофитную хуйню, когда на земле творится ад и израиль? Докопалися бы до ядра планеты тогда, нахуй ваши астероиды нужны?
495 484843
>>4842
В ядре нехуй делать.
А вот если по земле ебнет астероид в 100 километров - никого на планете живого не останется.

Достаточный ответ?
496 484845
>>4842

>непрофитную хуйню


Из очевидного - спутники-ретрансляторы для сигналов всех сортов, ЖПС. Дохуя чего неочевидного тож.
497 484846
>>4843

>А вот если по земле ебнет астероид в 100 километров - никого на планете живого не останется.


Если такая хуйня будет подлетать, то ее заметят за 5 наносек же

>>4845

>Из очевидного - спутники-ретрансляторы для сигналов всех сортов, ЖПС.


Это все рядом с землей. А нахуй дебилы лезут в другие галактики и тд?
498 484847
>>4846

>ее заметят за 5 наносек же


И нихуя не сделают.
Сейчас нет ничего что могло бы что-то с такой йобой сделать в принципе.
Останется только сидеть пару лет, молиться, поститься, слушать радио радонеж и смотреть на настоящий ад и израиль вокруг как люди совершают роскомнадзор и анархию.
499 484852
>>4817

>расстояние от Земли до Луны соизмеримо с размером Земли


Шобля, одно раз в 30 больше другого.
500 484859
>>4810

>Волны возникают за пределом горизонта событий, в чём парадокс?


С какого хуя они там возникают, если причина скрыта под горизонтом? Ведь нас в школе учили, что ничто из-под горизонта не способно повлиять на события внешней вселенной.
501 484901
>>4852
Двачую. Между луной и землей вместятся все планеты солнечной системы. Что тот анон хотел сказать?
502 484902
>>4859
Почему сам горизонт не может быть причиной?
504 484911
>>4909
Зачем так рано? До 550 можно сидеть и даже не тонуть.
505 484913
>>4911
Тред зашкварен горизонтоблядками, пусть они и тонут.
506 484914
>>4913
Тут такое дело - если в треде шитпост и зашквар, то тем более не надо катить, вдруг начальник опомнится и вилкой пошерудит и у нас 470 постов окажется.
Не раз такое было уже.

Ты бы честно признался, что спешил перекатить потому что придумал Мессье и не хотел чтобы кто-то другой по другому ученому перекатил, вопросов бы не было.
# OP 507 484926
>>4914
Не в этом дело. Во-первых, Мессье был заготовлен заранее; во-вторых, я был бы только за, если бы кто-то другой перекатывал, но за последние несколько тредов никто такого желания не выражал.

Просто мне кажется, что треды с самыми последними постами должны быть наверху, а не где-то внизу нулевой, чтобы сразу видеть сортировку по времени последнего обновления. Перекат сразу после бамплимита кажется мне естественным порядком вещей.
508 484927
>>4926
Окей, пусть будет так.
Ты годный ОП, достойная смена.
509 484958
>>4901
>>4852
Это соизмеримо, один порядок разницы. Сравни с расстоянием от Земли до Солнца. Солнце притягивает Землю в 1800 раз сильнее Луны, но приливов от Солнца нет. Почему? Именно потому, что Земля в масштабе расстояния Солнце-Земля - это точка и разницы в силе притяжения на разных ее сторона нет.
Тред утонул или удален.
Это копия, сохраненная 9 сентября 2019 года.

Скачать тред: только с превью, с превью и прикрепленными файлами.
Второй вариант может долго скачиваться. Файлы будут только в живых или недавно утонувших тредах. Подробнее

Если вам полезен архив М.Двача, пожертвуйте на оплату сервера.
« /spc/В начало тредаВеб-версияНастройки
/a//b//mu//s//vg/Все доски